#24 Rosh Review

¡Supera tus tareas y exámenes ahora con Quizwiz!

Question: What is a fatal complication of myasthenia gravis?

Answer: Respiratory failure. Rapid Review Myasthenia Gravis Patient will be complaining of proximal muscle weakness, ptosis, and diplopia that is worse at the end of the day PE will show ice test improves sx Diagnosis is made by edrophonium (tensilon) test, EMG Most commonly caused by autoimmune destruction of acetylcholine receptors Treatment is acetylcholinesterase inhibitors, such as pyridostigmine Comments: associated with thymoma

Question: What is the difference between Raynaud's disease and Raynaud's phenomenon?

Answer: Raynaud's phenomenon is the term used for Raynaud's disease when there is a specific causative etiology. These included scleroderma, rheumatoid arthritis and systemic lupus erythematosus. Rapid Review Raynaud's Disease Arteriolar vasospasm Triggered by cold, emotion Fingers become white → blue → red Rewarming, CCBs

Question: What is the most common manifestation of late Lyme disease?

Answer: Arthritis. Rapid Review Bell's Palsy Patient with a history of viral prodrome Complaining of waking up with unilateral facial nerve paralysis, hyperacusis and taste disturbance PE will show CN VII nerve palsy that does not spare the forehead Most commonly caused by HSV Treatment is prednisone, artificial tears, tape eyelid shut Comments: Bilateral: Lyme disease, infectious mononucleosis

Question: What is the definitive treatment for third-degree heart block?

Answer: Internal pacemaker placement.

Question: Does the volume of a pericardial friction rub increase or decrease with inspiration?

Answer: It increases during inspiration. Rapid Review Acute Pericarditis Idiopathic > viral (Coxsackie) Pleuritic chest pain radiating to the back Pain ↓ with leaning forward Pericardial friction rub ECG: diffuse STE, PR depression NSAIDs

Question: Adolescent boys with recurrent epistaxis and nasal obstruction might be the result of what benign but locally aggressive tumor?

Answer: Juvenile nasopharyngeal angiofibroma. Rapid Review Epistaxis Most common source: Anterior bleeds: Kiesselbach's plexus Posterior bleeds: Sphenopalatine artery Treatment is: Anterior bleeding: direct pressure, packing, cautery Posterior bleeding: packing (foley, gauze pack, intranasal balloon device) Admit patients with posterior packing to a monitored bed

Question: What is the normal level of methemoglobin in arterial blood?

Answer: Less than 2%.

Question: What are the two most common primary sources of spinal metastatic disease?

Answer: Lung and breast cancer.

Question: What is a common cause of bilateral peripheral facial nerve palsy?

Answer: Lyme disease. Rapid Review Bell's Palsy Patient with a history of viral prodrome Complaining of waking up with unilateral facial nerve paralysis, hyperacusis and taste disturbance PE will show CN VII nerve palsy that does not spare the forehead Most commonly caused by HSV Treatment is prednisone, artificial tears, tape eyelid shut Comments: Bilateral: Lyme disease, infectious mononucleosis

Question: What are the criteria for ultrasound diagnosis of pyloric stenosis?

Answer: Pyloric muscle thickness >4 mm and pyloric length > 14 mm. Rapid Review Pyloric Stenosis Patient will be 2 - 6 weeks old Complaining of non-bilious projectile vomiting after feeding and early satiety PE will show RUQ olive-like mass (hypertrophied pylorus) Labs will show hypochloremic hypokalemic metabolic alkalosis Diagnosis is made by ultrasound or UGI series (string sign) Treatment is surgical

Question: What is the gold standard test to diagnose obstructive sleep apnea?

Answer: An overnight polysomnography. Rapid Review Obstructive Sleep Apnea Patient will be obese With a history of allergies Complaining of apneic episodes while sleeping, snoring and daytime sleepiness PE will show enlarged tonsils Diagnosis is made by sleep study Treatment is weight loss, CPAP Comments: can lead to pulmonary hypertension and cor pulmonale (right ventricular hypertrophy)

Question: The disabling symptoms of premenstrual syndrome mentally or physically incapacitate what percentage of affected women?

Answer: 20-40%. Rapid Review Premenstrual Syndrome Onset: 1-2 weeks prior to cycle, resolves after menstruation begins Sleep disturbances ↓ Focus Emotional lability Breast tenderness, HA Does not hinder personal/professional life (unlike premenstrual dysphoric disorder) Initial rx: dec caffeine intake, exercise, stress reduction Pharmacologic rx: NSAIDs, SSRIs, OCPs

Question: What is the biological half-life of warfarin?

Answer: 36 to 42 hours.

Question: What is Levine's sign?

Answer: A clenched fist over the chest in patients with ischemic chest pain.

Question: What is the most common cause of an acute hemarthroses after a sports-related knee injury?

Answer: ACL tear. Rapid Review Anterior Cruciate Ligament (ACL) Tear Patient with a history of quickly stop moving and change direction while running Complaining of pop and swelling Diagnosis is made by Lachman's Test (most sensitive) and Anterior Drawer Test

Question: What are Ranson's five predictors of complications from acute pancreatitis upon admission?

Answer: Age >55 years, glucose >200, WBC >16,000, AST>250, LDH>350. Rapid Review Acute Pancreatitis Patient will be complaining of epigastric pain radiating to the back, nausea, and vomiting PE will show ecchymosis of left flank (GreyTurner sign), umbilical ecchymosis (Cullen sign) Labs will show elevated lipase (best) and amalyse Diagnosis is made by US and Ranson's criteria Most commonly caused by gallstones > alcohol Treatment is IV fluids

Question: What antibiotic is recommended as prophylaxis for high-risk patients undergoing a dental extraction?

Answer: Amoxicillin (2 grams by mouth). Rapid Review Endocarditis Patient will be complaining of fever, rash, cough and myalgias PE will show Fever, Roth spots, Osler nodes, Murmur, Janeway lesions, Anemia, Nailbed hemorrhages, Emboli (FROM JANE) Diagnosis is made by echocardiography and Duke's criteria Most commonly caused by: IVDA: S. aureus, tricuspid Native valve: Streptococci, mitral Treatment is antibiotics Comments: GI malignancy: S. bovis

Question: Which other antiplatelet medications are recommended for treating STEMI?

Answer: Aspirin, clopidogrel, ticagrelor and prasugrel. Rapid Review Acute Coronary Syndrome: Management Aspirin: ↓ mortality, ↓ infarct size, ↓ reinfarction rate Clopridogrel: patients with aspirin allergy Heparin: ↓ DVT, ↓ reinfarction, ↓ stroke, ↓ LV thrombus, ↓ reocclusion Nitroglycerin: Coronary artery dilation/vascular smooth muscle relaxation → ↓preload/afterload → ↓ myocardial O2 demand Contraindications: sildenafil use within 24 hrs, RV infarction ß-blockers: ↓ Myocardial O2 demand, ↓ ventricular fibrillation IV indications: tachydysrhythmias, intractable HTN Morphine: ↓ Preload/afterload, ↓ sympathetic activity No mortality benefit Glycoprotein IIb/IIIa inhibitors: benefit in patients undergoing PCI PCI: Preferred over thrombolytics in all STEMI patients PCI center: <90 minutes contact to device time Non-PCI center: transfer to PCI center if contact to device time can be <120 minutes Non-PCI center: thrombolytics if contact to device time to be >120 minutes Thrombolytics: begin within 30 minutes of ED arrival if selected

Question: What conditions make up the atopic triad?

Answer: Asthma, allergic rhinitis, atopic dermatitis.

Question: What is the role of beta-adrenergic blocking agents in acute myocardial infarction?

Answer: Beta-adrenergic blockers given within 24 hours of presentation reduce the risk of developing ventricular dysrhythmias. Rapid Review Cardiac Biomarkers Troponin Highest sensitivity and specificity Time detectable from onset: 3-12 hours Peak: 24-48 hours Return to baseline: 5-14 days CK-MB Time detectable from onset: 3-12 hours Peak: 24 hours Return to baseline: 48-72 hrs Useful for dx of reinfarction Myoglobin First to appear, first to peak, first to decline Lacks specificity

Question: What is the normal pressure of a tissue compartment?

Answer: Between 0-10 mm Hg.

Question: How is chronic kidney disease staged?

Answer: By estimating the glomerular filtration rate. Rapid Review Chronic Kidney Disease Definition: permanent loss of renal function > 3 months Most common cause: DM Uremic frost (urea crystals deposit on skin) Cardiovascular: HTN, pericarditis, CHF, atherosclerosis Volume overload/pulmonary edema ↓ EPO + Anemia of chronic disease → normocytic anemia: Hypovitaminosis D → 2° hyperparathyroidism → renal osteodystrophy Platelet dysfunction Waxy casts Lab findings: ↑ AG metabolic acidosis, ↑ phosphorus/potassium, ↓ calcium

Question: What is the most commonly isolated pathogen in patients with Guillain-Barré syndrome?

Answer: Campylobacter jejuni.

Question: What other gastrointestinal pathogen can give an appendicitis-like presentation?

Answer: Campylobacter jejuni. Rapid Review Yersinia Enterocolitica Patient with a history of eating undercooked pork Complaining of RLQ pain, fever, vomiting and then diarrhea Labs will show fecal WBCs and RBCs

Question: What are factors that promote sickling in sickle cell disease?

Answer: Cold, dehydration, hypoxia, infection, stress, menses, alcohol consumption. Rapid Review Sickle Cell Disease Sickling → vaso-occlusive ischemia ↓ O2, dehydration, acidosis → sickling Newborns: initially asymptomatic (due to ↑ HbF) Most common presentation in infants: dactylitis Aplastic crisis: ↓ Hb + reticulocytopenia, parvovirus B19 Acute chest syndrome Most common cause of death in adults Fever, CP CXR: pulmonary infiltrate Splenic sequestration crisis: rapid splenic sequestration of RBCs → splenomegaly + severe anemia Stroke Aseptic necrosis of the femoral head Dysfunctional spleen → ↑ infection risk Salmonella osteomyelitis S. pneumoniae sepsis: most common cause of death in children

Question: Is conventional angiography recommended for diagnosing renal artery stenosis?

Answer: Conventional angiography should be reserved for confirmation and therapeutic reasons, such as angioplasty and stent placement, because of the recent beneficial advances in the magnetic resonance angiography. Rapid Review Renal Artery Stenosis Causes: atherosclerosis, fibromuscular dysplasia Fibromuscular dysplasia: young females Refractory HTN Dx: doppler US, CTA Angioplasty Unilateral renal artery stenosis: ACEIs, ARBs

Question: What obstetrical complication occurs most often during the 6th and 8th week of pregnancy?

Answer: Ectopic pregnancies.

Question: When should cultures be obtained in conjunctivitis?

Answer: Failure of prior treatment or severe symptoms. Rapid Review Bacterial Conjunctivitis Patient will be complaining of red/pink eye with discharge usually worse in the morning PE will show purulent (yellow) discharge and crusting Most commonly caused by Staph aureus, Strep pneumoniae, H.Flu. Contact lens wearers: Pseudomonas aeruginosa Treatment is topical antibiotic drops

Question: True or false: The United States Preventive Services Task Force recommends that all men be screened for prostate cancer?

Answer: False.

Question: What is the Jarisch-Herxheimer reaction?

Answer: Fever, myalgias, and headache developing after penicillin therapy for syphilis. Rapid Review Primary Syphilis Patient will have been sexually active 2 - 3 weeks ago Complaining of bump on his penis PE will show painless, "Punched out" lesion Diagnosis is made by darkfield microscopy Most commonly caused by Treponema pallidum Treatment is single IM injection of benzathine penicillin Comments: syphiLIS is painLESS ulcer

Question: What is the antidote to benzodiazepine poisoning?

Answer: Flumazenil. Rapid Review Acetaminophen Toxicity: Patient will be complaining of ​abdominal pain, nausea, vomiting, and diaphoresis PE will show RUQ tenderness Labs will show elevated AST and ALT Treatment is N-acetylcysteine Comments: Rumack-Matthew nomogram - stratifies the risk of liver failure

Question: What are contraindications to getting rotavirus?

Answer: History of intussusception, weakened immune system, or life-threatening reactions to prior doses of the vaccine.

Question: What are the diagnostic criteria for preeclampsia?

Answer: Hypertension after 20 weeks of pregnancy plus proteinuria.

Question: What is the gold standard of treatment in slipped capital femoral epiphysis?

Answer: In situ pinning with a single, large screw. Rapid Review Slipped Capital Femoral Epiphysis (SCFE) Patient will be an obese male 12 - 16 years old Complaining of a progressive limp and knee pain PE will show loss of hip internal rotation Xray will show "scoop of ice cream slipping off an ice cream cone" Diagnosis is made by AP and frog-lateral X-rays Treatment is non-weight bearing and urgent orthopedic consultation

Question: Which medications can lead to lactic acidosis?

Answer: Metformin, salicylates, and propylene glycol. Rapid Review Metabolic Acidosis pH <7.35 + HCO3- <24 ​Normal AG: Hyperalimentation, Addison Disease, RTA, Diarrhea, Acetazolamide, Spironolactone, Saline infusion (HARDASS) ↑ AG: Methanol, Uremia, DKA, Propylene glycol, Iron or INH, Lactic acidosis, Ethylene glycol, Salicylates (MUDPILES) Tachypnea Winter's formula: PCO2 = 1.5(HCO3-) + 8 +/- 2 ​PCO2 < Winter's formula calculation = respiratory alkalosis PCO2 > Winter's formula calculation = respiratory acidosis

Question: What is another initial test done in any patient with increased vaginal discharge?

Answer: Microscopic examination of the secretions. Rapid Review Bacterial Vaginosis Patient will be complaining of malodorous vaginal discharge PE will show thin, gray/white discharge Labs will show pH > 4.5, clue cells Diagnosis is made by KOH to smear → fishy odor, "whiff test", Amsel Criteria Most commonly caused by Gardnerella vaginalis Treatment is metronidazole

Question: What is the most common cause of ventricular tachycardia?

Answer: Myocardial ischemia or infarct. Rapid Review Ventricular Tachycardia > 3 consecutive ectopic ventricular beats Monomorphic, polymorphic Bidirectional: digoxin toxicity Wide complexes Pulseless: immediate defibrillation Unstable: synchronized cardioversion Stable: procainamide, amiodarone, synchronized cardioversion (refractory) If unsure, manage all wide complex tachycardias as ventricular tachycardia

Question: In addition to lactulose, what is another treatment for hepatic encephalopathy?

Answer: Neomycin, Rifaxamin. Rapid Review Hepatic Encephalopathy Toxin accumulation due to liver dysfunction Causes: infection, dehydration, medications, GI bleed, hypoxia, hypoglycemia ↑ Ammonia CSF glutamine correlates with severity Classification system: Grade 1: mild cognitive dysfunction, irritability Grade 2: lethargy, disoriented, confusion asterixis Grade 3: somnolence inability to follow commands Grade 4: coma Rx: lactulose

Question: Are skin glues toxic to the eye?

Answer: No. Rapid Review Tissue Adhesives Rapidly applied Equivalent cosmetic result Resistance to bacterial growth Avoid high tension lacerations Use petroleum products to loosen bond

Question: Should beta-blockers be administered to lower the blood pressure in patients who have used cocaine?

Answer: No. This can lead to unopposed alpha-receptor stimulation and cardiovascular collapse. Rapid Review Cocaine Intoxication Sympathomimetic toxidrome Diaphoresis (unlike in anticholinergic overdose) Seizures Wide-complex dysrhythmias (rx: sodium bicarbonate) Rx: BZDs Avoid ßBs

Question: Which medications offer decent replacement therapy for hypoestrogenic women?

Answer: Oral contraceptive pills. Rapid Review Primary Amenorrhea Definition: lack of menarche by 16 with normal 2° sexual characteristics or by 14 with no 2° sexual characteristics Etiologies: Hypothalamic/pituitary Ovarian Uterine Pseudohermaphroditism Lab workup: FSH, LH, prolactin, TFTs, testosterone, hCG Rx: treat underlying cause

Question: What does the mini mental state examination test for?

Answer: Orientation, memory, attention. Rapid Review Dementia Gradual loss of mental capacity Preserved attention, motor function, speech Irreversible causes: Alzheimer's, vascular dementia, Creutzfeldt-Jakob disease Reversible causes: depression, B12 deficiency, syphilis, hypothyroidism, NPH, drug use, intracranial mass

Question: What other blood product is commonly transfused in a woman with abruptio placentae?

Answer: Packed red blood cells, to treat the volume-depleting and oxygen-carrying insult which stems from abruption. Rapid Review Placental Abruption Patient will be a pregnant women in her third trimester With a history of hypertension, trauma or cocaine use Complaining of painful vaginal bleeding Labs will show hypofibrinogenemia

Question: What are the characteristic findings in aplastic anemia?

Answer: Pancytopenia and a hypocellular bone marrow.

Question: What is the main contraindication to a dental nerve block?

Answer: Patient history of hypersensitivity or allergic reaction to local anesthetic agents. Rapid Review Facial and Dental Nerve Blocks Supraorbital: ipsilateral forehead and scalp Infraorbital: area between lower eyelid and upper lip Posterior superior alveolar: ipsilateral maxillary molars Inferior alveolar: ipsilateral mandibular teeth, lower lip, chin Mental: ipsilateral lower lip and chin

Question: What is the most common cause of nonvariceal upper GI bleeding?

Answer: Peptic ulcer disease.

Question: Which class of BPH medications is most likely to improve voiding as well as any concomitant erectile dysfunction?

Answer: Phosphodiesterase-5 inhibitors are FDA approved to manage urinary symptoms and erectile dysfunction. Rapid Review Benign Prostatic Hyperplasia (BPH) Patient will be older Complaining of Hesitancy, Intermittence/Incontinence, Frequency/Fullness, Urgency, Nocturia (HI FUN) PE will show a soft, smooth and mobile prostate without any nodules or indurations Diagnosis is made by digital rectal exam Most commonly caused by stromal and epithelial cell growth in the transitional zone of the prostate Treatment is α-blockers, 5α-reductase inhibitors, surgery (TURP)

Question: Why should patients with posterior nasal packs be admitted to a monitored bed?

Answer: Posterior packing can cause vagal stimulation with resulting bradycardia and bronchoconstriction. Rapid Review Epistaxis Most common source: Anterior bleeds: Kiesselbach's plexus Posterior bleeds: Sphenopalatine artery Treatment is: Anterior bleeding: direct pressure, packing, cautery, Posterior bleeding: packing (foley, gauze pack, intranasal balloon device) Admit patients with posterior packing to a monitored bed

Question: What are the most common causes of acute urinary obstruction in women?

Answer: Prolapse of pelvic organs and pelvic tumors. Rapid Review Benign Prostatic Hyperplasia (BPH) Patient will be older Complaining of Hesitancy, Intermittence/Incontinence, Frequency/Fullness, Urgency, Nocturia (HI FUN) PE will show a soft, smooth and mobile prostate without any nodules or indurations Diagnosis is made by digital rectal exam Most commonly caused by stromal and epithelial cell growth in the transitional zone of the prostate Treatment is α-blockers, 5α-reductase inhibitors, surgery (TURP)

Question: Why should fluoroquinolones be withheld in the treatment of patients with pneumonia and a suspicion for TB?

Answer: Respiratory fluoroquinolones are among the medications that may be used in the treatment of TB and may alter the results of sputum samples for acid-fast bacilli. Rapid Review Systemic Lupus Erythematosus (SLE) African-Americans, females Malar rash Arthritis Renal disease Cardiac: fibrinous pericarditis, Libman-Sacks endocarditis CNS: HA, stroke, seizures Drug induced: Hydralazine, INH, Procainamide, Phenytoin, Sulfonamides (HIPPS) Antinuclear antibodies (ANA): 100% sensitive, not specific Anti-dsDNA antibodies: 100% specific, poor prognosis Anti-Smith antibodies: 100% specific, not prognostic Antihistone antibodies: sensitive for drug-induced lupus NSAIDs, steroids, immunosuppressants, hydroxychloroquine

Question: Where are most aspirated foreign bodies located?

Answer: Right main bronchus. Rapid Review Foreign Body Aspiration Patient will be a child Complaining of a sudden episode of coughing, wheezing, or stridor Comments: most common is the right main bronchus

Question: What treatment should be given to colonized household contacts of children with tinea capitis?

Answer: Selenium sulfide shampoo.

Question: Which other condition mimics Osgood-Schlatter disease?

Answer: Sinding-Larsen-Johansson syndrome, a similar pathology in the inferior patellar pole of boys aged 9-11 years. Rapid Review Osgood-Schlatter Disease Patient will be a boy athlete 10 - 15 years old Complaining of knee pain while running PE will show tenderness over the tibial tubercle Treatment is ice, NSAIDs, quadriceps stretching

Question: What has a greater risk of infection, cutaneous or subcutaneous sutures?

Answer: Subcutaneous.

Question: Compare the timing of aortic valve murmurs?

Answer: Systolic = stenosis, diastolic = regurgitation. Rapid Review Aortic Stenosis Patient will be older With a history of diabetes, hypertension Complaining of dyspnea, chest pain, syncope PE will show crescendo-decrescendo systolic murmur that radiates to the carotids, paradoxically split S2, S4 gallop Most commonly caused by degenerative calcification Treatment is aortic valve replacement Comments: murmur decreases with Valsalva

Question: What is the estimated mortality rate in undiagnosed Brugada syndrome?

Answer: Ten percent per year.

Question: How does ulcer pain correlate with ulcer location?

Answer: The pain from duodenal ulcers is usually relieved by food, while pain from a gastric ulcer worsens after food intake. Rapid Review Peptic Ulcer Disease Most common cause of UGIB RFs: smoking, H. pylori, NSAIDS, ASA, steroids Duodenal > gastric Duodenal: pain decreases food Gastric: pain greater with food Proton pump inhibitors, endoscopy Complications: perforation, gastric outlet syndrome

Question: How do NSAIDs relieve the pain of dysmenorrhea?

Answer: Their antiprostaglandin action counters the underlying pathophysiology. Rapid Review Primary Dysmenorrhea Occurs only during menstrual cycles ↑ PGF2α → ↑ uterine contractions NSAIDs, acetaminophen

Question: What type of vaccine is the varicella-zoster virus vaccine?

Answer: There are currently 2 different VZV vaccine preparations. Both are live-attenuated viruses and recommended for all immunocompetent infants over one year of age and all noninfected immunocompetent adults. Rapid Review Varicella Viral prodrome → maculopapular rash → clear vesicles on an erythematous base ("dew drop rash on a rose petal") Lesions occur in crops Contagious 5 days before and after vesicles Multiple stages of lesions present Treatment: <12: supportive care >12: acyclovir Immunocompromised: IV acyclovir

Question: Which gender is more likely to experience all 5 stages of grief?

Answer: Women are more likely than men to experience all 5 stages. Rapid Review Grief Reaction Stages: denial, anger, bargaining, depression, acceptance Nonpathologic symptoms: guilt, weight loss, simple hallucinations Pathologic symptoms: >12 months, severe functional impairment, suicidal ideation, psychotic symptoms

Which of the following treatments has a proven mortality benefit in ST-elevation myocardial infarction? Aspirin Morphine Nitroglycerin Oxygen

Correct Answer ( A ) Explanation: ST elevation myocardial infarction (STEMI) occurs when there is thrombus formation causing a blockage of a coronary vessel. Common symptoms include gradual onset of chest tightness or pressure that radiates to the arms or neck. The classic teaching is to give morphine, nitroglycerin, oxygen, and aspirin for acute coronary syndrome. However, only aspirin has shown a mortality benefit. In recent prospective studies morphine (B) has been associated with a higher mortality rate but no causative relationship has been established. Although nitroglycerin (C) can help reduce ischemic pain through coronary vasodilation it does not reverse the underlying cause. It is not associated with a mortality benefit. Supplemental oxygen (D) is not associated with a mortality benefit and may even cause harm in non-hypoxic patients.

A woman with chest pain presents to the ED. Her electrocardiogram shows ST elevation in leads V3 and V4. She is started on oxygen, intravenous nitroglycerin, and aspirin. You are preparing her for transfer to the interventional cardiac unit for primary percutaneous cardiac reperfusion. Her heart rate has been consistently < 60, averaging 54 over the past 30 minutes. Which of the following medications is also appropriate treatment at this time? Abciximab Infliximab Metaproterenol Metoprolol

Correct Answer ( A ) Explanation: ST-elevation myocardial infarction in the anterior heart is evidenced by chest pain and ECG changes as above. The most likely site of complete arterial thrombus in this scenario is the left anterior descending artery. In a percutaneous coronary intervention (PCI) capable hospital, the goal is to get the patient to PCI within 90 minutes. Once this is possible, it is recommended to administer a GP IIb/IIIa inihibitor, such as abciximab, eptifibatide or tirofiban. Some research has shown a 60% decrease in death and further MI with this class of medications. Abciximab is a platelet aggregation inhibitor mainly used during and after coronary artery procedures like angioplasty to prevent platelets from sticking together and causing thrombus formation within the coronary artery. The use of abciximab in this setting is associated with a decreased incidence of ischemic complications due to the procedure and a decreased need for repeated coronary artery revascularization in the first month following the procedure. Infliximab (B) is an anti-tumor necrosis factor alpha antibody used in the treatment of autoimmune disease, not STEMI. Beta-2 agonists, such as metaproterenol (C), are used in the acute management of asthmatic bronchospasm, not STEMI. Beta-blockers, such as metoprolol (D), do have an anti-arrhythmic benefit in STEMI, however, their use is contraindicated if pulse is < 60 bpm or systolic blood pressure is < 120 mm Hg.

A three-year-old boy is brought to the emergency department due to acetaminophen ingestion. About two hours prior, he was found by his mother with an open bottle of acetaminophen and several tablets scattered on the floor. The mother was not sure how many tablets were missing. The boy had one episode of vomiting at home. He is asymptomatic upon arrival at the emergency room. On examination, the boy is active with normal vital signs and physical examination. Which of the following is the most appropriate next step in management? Get acetaminophen level four hours after ingestion Obtain transaminases, synthetic and renal function Perform immediate gastric lavage Treat with syrup of ipecac urgently

Correct Answer ( A ) Explanation: Acetaminophen is commonly present in the home, which can be unintentionally ingested by young children. The initial signs of acetaminophen toxicity are nonspecific such as nausea and vomiting, which is usually followed by an asymptomatic period. Any child with a history of acute ingestion of greater than 200 mg/kg should be referred to a healthcare facility. If a toxic ingestion is suspected, a serum acetaminophen level should be calculated four hours after the reported time of ingestion. For patients who present to medical care more than four hours after ingestion, a stat acetaminophen level should be obtained. Acetaminophen levels obtained fewer than four hours after ingestion are difficult to interpret and cannot be used to estimate the potential for toxicity. The serum acetaminophen level is then plotted on the Rumack-Matthew nomogram and any level that is in the possible or probable hepatotoxicity range should then be treated with N-acetylcysteine. Obtaining transaminases, synthetic and renal function (B) can be done for baseline, and these laboratory values followed serially in cases of acetaminophen ingestion. However, it is more important to obtain an acetaminophen level to direct management. Performing immediate gastric lavage (C) involves placing a tube into the stomach to aspirate contents, followed by flushing with aliquots of fluid. Although it was used routinely in the past, objective data do not document or support clinically relevant efficacy. Thus in most cases, its use is no longer recommended. Treating with syrup of ipecac urgently (D) would act on the central nervous system and gastrointestinal system to produce vomiting. After thorough evidence review, it is no longer recommended by the American Academy of Pediatrics.

A 22-year-old man presents to the Emergency Department after a syncopal episode. His ECG is shown above. He is currently asymptomatic with normal vital signs. Which of the following is the most appropriate next step in management? Admit to a telemetry unit for cardiology consult and AICD placement Discharge with cardiology follow up as an outpatient Order emergent echocardiogram Start amiodarone for treatment of presumed ventricular tachycardia

Correct Answer ( A ) Explanation: Brugada syndrome was initially described in 1992 and is an inherited disorder that is the result of a mutation in the cardiac sodium channel gene. Patients are predisposed to malignant dysrhythmias and sudden death. The mean age of sudden death is 41 years and males are significantly more likely than females to be affected. Most patients are asymptomatic and found only by incidental ECG. The type 1 ECG pattern characteristic of Brugada syndrome is coved ST segment elevation of greater than 2 mm in V1-V3. It can be transient and may be augmented by fever or pharmacologic stimulation (particularly sodium channel blockers). The type 1 pattern is considered diagnostic when combined with clinical criteria such as syncope, family history of early sudden death, characteristic ECG changes in family members or history of ventricular fibrillation or ventricular tachycardia. The type 2 ECG pattern of saddleback ST segment elevation in V1-V3 is suggestive of Brugada syndrome but not diagnostic. Management involves admission to a telemetry unit for cardiology consultation. Definitive treatment is implantable cardioverter defibrillator placement. Patients with characteristic ECG changes and clinical criteria such as syncope are at a high risk for malignant dysrhythmias and sudden death and should not be discharged (B). There is no structural defect associated with Brugada syndrome so an echocardiogram (C) is not indicated in the the workup. Medications such as amiodarone (C) do not play a role in the management of Brugada syndrome. The only proven therapy to prevent sudden death in these patients is AICD placement.

Which of the following patients is most likely suffering from dementia? A confused 65-year-old man brought from a house fire who exhibits good long-term recall, no recall of immediate events, normal vital signs, carboxyhemoglobin level of seven A confused 65-year-old man brought from a house fire who exhibits poor long-term recall, no recall of immediate events, normal vital signs, carboxyhemoglobin level of 25 A confused 80-year-old man brought from home with fever, back pain, and urinary retention A confused 80-year-old man brought from home with fever, cough, and hypoxia A confused 80-year-old man brought from home with increased thirst, increased urination, and blood sugar monitor that reads "high"

Correct Answer ( A ) Explanation: Dementia results from a gradual loss of mental capacity with preservation of motor and speech. Dementia generally occurs in the elderly. Remote memories are often preserved. The most common types of dementia are Alzheimer's disease and vascular dementia, both insidious in onset. Symptoms may worsen acutely because of a concurrent medical condition. Physical exam and laboratory workup are generally unrevealing. This patient has acute delirium (B) caused by carbon monoxide poisoning (CO). CO poisoning can lead to confusion, headache, vomiting, and syncope (C, D, E). A febrile illness can lead to delirium in elderly patients. In the evaluation of all such patients, it is critical to differentiate acute delirium, which is a confusional state where functions of cognition and attention are impacted due to an underlying medical cause from chronic dementia.

A 39-year-old woman presents to the ED with sharp pain of the right lower second premolar (tooth #29). On exam, you note a dental cavity with no surrounding erythema or fluctuance. Which of the following nerve blocks is most appropriate for this patient? Inferior alveolar nerve block Infraorbital nerve block Posterior superior alveolar nerve block Supraperiostal nerve block

Correct Answer ( A ) Explanation: Facial and dental blocks are used to provide temporary analgesia for intraoral and dental pain related to facial trauma or dental infection. The inferior alveolar nerve block anesthetizes all mandibular teeth to the midline, the anterior two thirds of the tongue, and the floor of the oral cavity. The infraorbital nerve block (B) provides anesthesia to the maxillary teeth from the midline through the canine, the buccal soft tissue of the upper lip, the lateral aspect of the nose, and the lower eyelid. The posterior superior alveolar nerve block (C) anesthetizes the second and third maxillary molars and—in 70% of patients—the first maxillary molar. The supraperiosteal nerve block (D) anesthetizes individual maxillary teeth but works poorly on mandibular teeth due to the increased bone density of the mandible.

A 65-year-old man presents with acute urinary retention. What is the most common cause of acute urinary retention in this patient? Benign prostatic hyperplasia Urethral stricture Urinary calculus Urinary tract infection

Correct Answer ( A ) Explanation: In men over the age of 50 years of age, benign prostatic hyperplasia (BPH) is the most common cause of acute urinary retention (AUR). AUR is the sudden inability to void urine voluntarily. It is most commonly caused by obstructive lesions. BPH is the most common precipitant and is implicated in about 50% of patients. Urinary output is obstructed by enlargement of the prostate and constriction of the prostatic urethra (from alpha-adrenergic tone). Approximately 25% of men with AUR will have prostate cancer instead of BPH. A urinary calculus (C) in the urethra can cause AUR. Urinary tract infections (D) are less common in men and rarely causes them to become obstructed. Other less common causes include urethral strictures (B) and phimosis.

A 65-year-old man presents to your office with complaints of urinary frequency, hesitancy and nocturia. Which of the following findings would suggest a diagnosis of prostate cancer rather than benign prostatic hypertrophy? Asymmetric areas of induration on digital rectal exam Elevated serum prostate specific antigen Presence of hematuria on urinalysis Symmetric enlargement and firmness of the prostate on digital rectal exam

Correct Answer ( A ) Explanation: In the United States, prostate cancer is the most frequently diagnosed type of cancer in men after skin cancer. Prostate cancer is seen more commonly in African-American men and the likelihood of developing this type of cancer increases with age. It is uncommon in men younger than 50 years of age. Risk factors include a family history of prostate cancer, cigarette smoking and a diet high in animal fat. Most patients diagnosed with prostate cancer are asymptomatic and the cancer is found on digital rectal exam (DRE) or because of an elevated serum prostate specific antigen (PSA). A physical exam finding of asymmetric areas of induration or nodules on DRE is suspicious for prostate cancer and further workup should be initiated. Diagnosis is made with biopsy. Treatment decisions are determined after discussion with the patient about the severity of disease and quality of life related to treatment side effects. Treatment options include active surveillance, prostatectomy, radiation therapy and hormonal therapy. Elevated serum prostate specific antigen (B) and the presence of hematuria on urinalysis (C) may be seen in both prostate cancer and benign prostatic hyperplasia (BPH). Physical exam findings help to differentiate between the two conditions. The diagnostic approach to patients with suspected BPH includes complete history, physical exam, urinalysis, serum PSA and creatinine, and evaluation for urinary obstruction. Symmetric enlargement and firmness of the prostate on DRE (D) is a physical finding seen in patients with BPH.

A 25-year-old woman presents to your office with amenorrhea, abdominal cramping, and irregular vaginal bleeding. She usually has monthly menstrual cycles, but has been intermittently spotting over the last 2 months. Her exam shows a firm uterus without tenderness and a dilated cervix with blood in the vaginal vault. Her beta HCG is positive and a bedside ultrasound reveals a yolk sac, fetal pole and no evidence of cardiac activity. What is the most likely diagnosis? Inevitable abortion Physiologic bleeding Septic abortion Threatened abortion

Correct Answer ( A ) Explanation: Inevitable abortion is associated with painful abdominal cramps, vaginal bleeding and a dilated cervix with gestational tissue often visible at the cervical os or on ultrasound. Dilatation and curettage is indicated for significant cramping or blood loss. First-trimester miscarriage is a common event. All women with clinically recognized pregnancies have approximately a 15% chance of spontaneous pregnancy loss in the first 3 months of pregnancy. There are many causes of spontaneous pregnancy loss in the first trimester; sporadic chromosomal aneuploidy accounts for about 60%. The remaining 40% have various etiologies, including chronic maternal illness (e.g., diabetes, connective tissue disorder), uterine structural malformations, certain infections, inadequate progesterone production, and possibly other, less-well-understood causes such as immunologic "rejection" and environmental factors. Physiologic bleeding (B) is vaginal spotting that occurs early in the first trimester with no cramping or other symptoms with a closed cervix on exam. Septic abortion (C) is diagnosed when there is infection of the uterus and products of conception. This occurs most often with incomplete abortions. Fever, uterine tenderness, foul discharge, and leukocytosis are often present. Threatened abortion (D) is diagnosed when the pregnant woman presents with vaginal bleeding, lower back discomfort, or midline pelvic cramping. On examination, the cervix is closed, and the pregnancy is viable.

A 17-year-old girl with a history of asthma presents to your office with complaints of wheezing and shortness of breath. She says that in the past month she has experienced symptoms approximately 3-4 times per week requiring use of her short-acting beta agonist inhaler. She has also woken up at night four times during the month with shortness of breath and occasionally gets dyspneic on her daily walk. Which of the following is the most effective management? Add a low dose inhaled glucocorticoid Add a low dose inhaled glucocorticoid plus long-acting beta agonist Continue use of short-acting beta agonist only Start an oral course of glucocorticoids

Correct Answer ( A ) Explanation: Mild persistent asthma is defined as having symptoms more than twice per week but less than daily, 3-4 nocturnal awakenings per month, symptoms requiring the use of a short-acting beta agonist rescue inhaler more than two times per week, minor interference with activities of daily living, FEV1 measurements and FEV1/FVC ratio within normal limits and two or more asthma exacerbations requiring an oral course of corticosteroids per year. Treatment of asthma follows a step-wise plan depending on the severity of symptoms. Mild persistent asthma requires daily use of a long-term controlling medication and a low dose inhaled glucocorticoid. Use of a short-acting beta agonist rescue inhaler should also continue as needed. Low dose inhaled glucocorticoid plus a long-acting beta agonist (B) is the recommended treatment for patients with moderate persistent asthma. A medium dose inhaled corticosteroid may also be used instead of using two medications. Use of a short-acting beta agonist only (C) is indicated for patients with mild intermittent asthma. The use of a rescue inhaler more than two times per week is characteristic of mild persistent asthma. An oral course of glucocorticoids (D) is part of step six asthma treatment for asthma exacerbations. Consultation with an asthma specialist should occur for patients requiring step four treatment or higher.

A 45-year-old woman uses ibuprofen daily for the last 12 months to help with her chronic back pain. She comes to you with new onset epigastric pain. Endoscopic examination reveals a duodenal ulcer. Helicobacter pylori serologic testing is positive. She has no known drug allergies. In addition to stopping the ibuprofen, which of the following is the most appropriate therapy at this time? Clarithromycin plus amoxicillin plus a proton pump inhibitor Clarithromycin plus an H2-antagonsist Metronidazole plus amoxicillin plus bismuth subsalicylate Metronidazole plus bismuth subsalicylate

Correct Answer ( A ) Explanation: Peptic ulcer disease has a lifetime prevalence of 10%. It has many etiologies, including H.pylori infection, NSAIDs, aspirin, bisphosphonates, gastric hypersecretory states, gastric cancer, tobacco, psychological stress, chemo and radiotherapy and infection with cytomegalovirus and herpes simplex virus. Half of the American population is colonized with H.pylori, but only 5-10% develop ulcers from it. 80% of duodenal ulcers and 60% of gastric ulcers are due to H.pylori. Ulcers mostly present with epigastric abdominal pain. However, patients may present with the main complications of peptic ulcer disease, namely upper GI bleeding from perforation or penetration. Diagnosis requires endoscopy. All gastric ulcers require biopsy to rule-out malignancy. Tests for H. pylori include serology, stool antigen and rapid urease testing. Treatment includes H. pylori eradication if present, lifestyle changes, NSAID and aspirin cessation and proton pump inhibitors (PPI). Surgery may be required if patients are refractory to medication management or perforation/penetration is present. Treatment of H.pylori-positive peptic ulcer disease is the classic triple therapy of clarithromycin + amoxicillin + PPI, all given twice a day for 10-14 days. Single antibiotic therapy (B and D) is insufficient. H2-antagonists (B) are not considered in the treatment of peptic ulcer disease. Bismuth subsalicylate is always used in combination with a PPI for treating peptic ulcer disease. If a patient has an amoxicillin allergy, or if clarithromycin resistance is suspected, then quadruple therapy is prescribed. This consists of metronidazole + tetracycline + bismuth + PPI, not metronidazole and amoxicillin (C). Sequential treatment is emerging as a beneficial option. It consists of an antibiotic plus PPI for 5 days, then 2 different antibiotics plus PPI for 5 days.

The presence of which of the following best differentiates premenstrual syndrome from premenstrual dysphoric disorder? Anger and irritability Confusion and other cognitive changes Mood swings and other mood changes Serotonin dysfunction

Correct Answer ( A ) Explanation: Premenstrual syndrome (PMS) is not clearly defined, as its high prevalence, unclear etiology and myriad of symptoms makes it difficult to classify it as a disease or a cluster of physiologic changes. Age of presentation is mainly in the late 20s to early 30s, and it recurs in up to 75% of affected women. Etiological theories include psychological disturbance, alterations in estrogen and progesterone balance as well as serotonin function, hypoglycemia and hyperprolactinemia. Symptoms include headache, insomnia, fatigue, low energy, bloating, breast tenderness, abdominopelvic pain, depression, anxiety, dysphoria, mood lability, appetite changes, crying episodes, confusion, poor coordination and poor concentration. Symptoms typically interfere with the woman's daily life. According to the American Psychiatric Association DSM-IV, prominent anger, irritability and internal tension associated with severe premenstrual syndrome symptoms is defined as premenstrual dysphoric disorder. Confusion and other cognitive changes (B), mood swings and other mood changes (C), and serotonin dysfunction (D) are all associated with premenstrual syndrome. The presence of prominent anger, irritability and internal tension in combination with premenstrual symptoms differentiates premenstrual dysphoric disorder from premenstrual syndrome.

A 17-year-old man presents with a "bump" on his penis. He denies other symptoms, significant past medical history, or recent travel. He has had a new sexual partner for the past three weeks, but she has not had reported symptoms. Exam reveals an erythematous but non-tender papule with slight ulceration at its center. What is the treatment of choice? Intramuscular benzathine penicillin G Intramuscular ceftriaxone Oral acyclovir Oral azithromycin in a single dose

Correct Answer ( A ) Explanation: The patient has the typical presentation of a chancre, which is caused by Treponema pallidum. A painless chancre is the hallmark of primary syphilis and appears two to three weeks after inoculation with bacteria. In immunocompetent hosts, it begins as a single papule, which then ulcerates at its center, leaving behind raised edges. The ulcer is typically non-exudative. Many patients also develop bilateral inguinal lymphadenopathy. Because the chancre is painless, it may not be noticed by the patient or brought to medical attention prior to self-resolution. Untreated, the patient will transition into latent syphilis and has a 25% chance of developing symptoms of secondary syphilis. The treatment of choice for primary syphilis is intramuscular benzathine penicillin G. Benzathine penicillin remains detectable in serum for up to thirty days after administration. Treatment with shorter-acting penicillins are associated with treatment failure and thus is not recommended. Intramuscular ceftriaxone (B) has not been well-studied in the treatment of syphilis but may be an alternative for patients with non-immediate-type hypersensitivity reactions to penicillin. Ceftriaxone is the treatment of choice for Neisseria gonorrhoeae, which most commonly causes urethritis in males. Genital ulcers are not associated with N.gonorrhoeae. Oral acyclovir (C) is the treatment of choice for genital herpes, which is characterized by recurrent outbreaks of genital vesicles that ulcerate into painful, shallow ulcers. The above patient's single, painless ulcerated papule make genital herpes infection unlikely. Oral azithromycin in a single dose (D) is a second-line treatment for syphilis, but it is not as effective as penicillin. Notably, pregnant women should never receive second-line syphilis therapy even if allergic to penicillin. Azithromycin is also the treatment of choice for C.trachomatis. In the US, C.trachomatis typically causes urethritis and cervicitis, not genital ulcers. However, C.trachomatis serovars L1-L3 are prevalent in tropical and subtropical areas of the world and cause lymphogranuloma venerum (LGV). LGV is characterized by development of a painless genital ulcer that self-resolves prior to development of marked inguinal lymphadenopathy. As the patient has not recently traveled and the ulcer appears typical of chancre, syphilis is more likely than LGV in the above patient.

A 18-year-old man presents to the ED in police custody after using an unknown drug. Vital signs are BP 170/85, HR 120, T 37.8°C, RR 18, and pulse ox 99% on room air. On exam, he is agitated and diaphoretic. His pupils are 7 mm and reactive. Which of the following substances did this patient most likely use? Cocaine Dextromethorphan Fluoxetine Phencyclidine

Correct Answer ( A ) Explanation: The sympathomimetic toxidrome is seen with the acute abuse of cocaine, amphetamines, or decongestants. Cocaine causes release of dopamine, epinephrine, norepinephrine, and serotonin. The greatest impact comes from the adrenergic stimulation by norepinephrine and epinephrine. Norepinephrine causes vasoconstriction by stimulating alpha-adrenergic receptors on vascular smooth muscle. Epinephrine increases myocardial contractility and heart rate through stimulation of beta-1-adrenergic receptors. In addition to causing catecholamine release, the reuptake of these neurotransmitters is inhibited. Clinically, patients are usually hypertensive and tachycardic and exhibit mydriatic pupils. In massive overdoses, cardiovascular collapse can result in shock and wide-complex dysrhythmias. CNS effects include seizures. Sympathomimetic toxidrome is sometimes difficult to distinguish from anticholinergic toxidrome. The difference is that patients usually present with dry mucous membranes with an anticholinergic overdose, whereas patients are diaphoretic with sympathomimetics. Treatment is usually supportive. Benzodiazepines should be administered for agitation, hypertension, and hyperthermia. Dextromethorphan (B), a common ingredient in cold preparations, shares similar properties to phencyclidine (PCP) and other opioid compounds. It inhibits the uptake of serotonin and blocks the NMDA receptor at the PCP binding site. Clinically, overdose causes lethargy, agitation, dysarthria, ataxia, diaphoresis, hypertension, and nystagmus. At high doses, intoxication resembles that of LSD, with euphoria and hallucinations. Dystonic reactions have been reported in children after therapeutic administration. Because dextromethorphan inhibits the uptake of serotonin, when it is ingested with a serotonin reuptake inhibitor, it can induce serotonin syndrome. Fluoxetine (C) is a selective serotonin reuptake inhibitor (SSRI). In overdose, the increased serotonin has its greatest affect on the gastrointestinal (nausea, vomiting, abdominal pain); cardiovascular (tachycardia, bradycardia, QTc prolongation); and central nervous systems (dizziness, hyper-reflexia, agitation). Overdose of fluoxetine alone rarely produces serotonin syndrome. Phencyclidine (PCP) (D) is a dissociative agent that when smoked, leads to a wide variety of findings. Patients may exhibit bizarre behavior, agitation, and violence. A blank or catatonic stare is common. Vertical, horizontal, and rotary nystagmus is often present.

Which of the following is true regarding the use of tissue adhesives for wound repair? Frequent prolonged exposure to water should be avoided to prevent premature breakdown of the tissue adhesive Standard wound care, including use of topical antibiotic ointment, should be performed on lacerations closed with tissue adhesives Tissue adhesives can be used to close lacerations on mucosal surfaces Tissue adhesives can be used to close lacerations that occur as a result of animal bites

Correct Answer ( A ) Explanation: Tissue adhesives can break down prematurely with frequent prolonged exposure to water. Therefore patients should be instructed to avoid soaking the affected area and activities such as swimming. Patients can shower and pat the area dry, however scrubbing and vigorous washing should be avoided. Tissue adhesives, or skin glues, are cyanoacrylates formulated for use on human skin. They are indicated for closure of linear, non-contaminated lacerations to the face, extremities and torso. Generally they are effective on wounds with little tension that would be amenable to repair with 5-0 sized suture material. Benefits of tissue adhesives include: ease and speed of application, minimal to no patient discomfort with application, reduction in needle-stick risk to healthcare providers, provides a protective covering to the wound and eliminate need for follow-up for suture removal. Cosmetic result is similar if not better than repair with sutures. Wound strength is initially less with skin glue closure, however after 7 days is comparable to sutured wounds. Tissue adhesive breaks down with in 5-10 days and the material will slough off without intervention. Contraindications to the use of skin glue include: jagged or irregular-shaped lacerations; wounds sustained as a result of an animal or human bite (D), puncture or crush injury; contaminated or infected wounds; wounds on mucosal surfaces (C); wounds in high moisture areas (axilla, perineum, palms); wounds overlying joints (unless they are immobilized). Tissue adhesives are broken down by the application of ointments (B), such as triple antibiotic or bacitracin. Therefore patients should be instructed to avoid putting these on lacerations that are repaired with skin glue. If removal is needed, antibiotic ointment and petroleum jelly are effective in aiding in removal.

A woman in her third trimester of pregnancy is involved in a motor vehicle collision. She presents to the ED with new onset vaginal bleeding and pelvic pain. Which of the following laboratory abnormalities would you expect to find in this patient? Decreased prothrombin time Hypofibrinogenemia Proteinuria Thrombocytosis

Correct Answer ( B ) Explanation: Abruptio placentae is a condition of premature separation of the placenta from the uterus. This patient exemplifies the presentation of placental abruption. Abnormal placenta-uterus separation may lead to significant fetal and maternal stress. One of the most common maternal complications is a consumptive coagulopathy. Placental separation results in intravascular and retroplacental coagulation. This excessive coagulation depletes platelets, fibrinogen and other clotting factors, leading to thrombocytopenia and hypofibrinogenemia, as well as an increase in the INR and the activated partial thromboplastin time. If placental abruption is a suspected cause of third trimester bleeding, laboratory evaluation of the above values should be obtained early in the management plan. If abnormalities are found, component therapy should be initiated via transfusions of platelets and fresh frozen plasma. there is an increase in prothrombin time (A) and thrombocytopenia (D). Proteinuria (C) is a hallmark abnormality in the late pregnancy hypertensive condition preeclampsia.

Bilateral facial nerve palsy in children is pathognomonic for which of the following conditions? Guillian-Barré Lyme disease Myasthenia gravis Systemic lupus erythematosus

Correct Answer ( B ) Explanation: Bilateral facial nerve palsy is virtually pathognomonic for Lyme disease in children. Cranial nerve palsy of cranial nerve VII (facial nerve) is a common manifestation of Lyme disease in children. Factors associated with Lyme disease include fever, headache, and absence of history of herpetic lesions. The palsy in Lyme disease may be a result of mononeuritis multiplex, which is a peripheral neuritis, or of basilar meningitis with involvement of the facial nerve. Facial nerve palsy associated with Lyme disease usually resolves completely with or without antimicrobial therapy. Antimicrobial therapy is necessary to prevent other manifestations of Lyme disease, including late cardiovascular and neurological complications. Guilian-Barré (A) is associated with ascending paralysis. Myasthenia gravis (C) is associated with ptosis, which is the inability to open the upper eyelids. Systemic lupus erythematosus (D) is associated with a facial malar rash. Bilateral nerve palsy is not pathognomonic for any of these conditions.

Epistaxis most commonly occurs in which location? Anterior nasal cavity at inferior turbinates Anterior nasal cavity at Kiesselbach's plexus Posterior nasal cavity at adenoids Posterior nasal cavity at superior turbinates

Correct Answer ( B ) Explanation: Epistaxis is classified according to its location. Bleeding from the anterior nasal cavity is most common and usually originates from a rich plexus of vessels at the anterior septum called Kiesselbach's plexus. Bleeding from this location, although troublesome, is less likely to be severe and is usually easier to control than posterior epistaxis. Predisposing factors for epistaxis include trauma (facial trauma or self-inflicted digital trauma), dry weather, hypertension, bleeding dyscrasias (factor deficiencies, hereditary hemorrhagic telangiectasia, lymphoproliferative disorders), anticoagulation therapy (acetylsalicylic acid, heparin, warfarin), and intranasal tumors. Anterior nasal cavity is the most common location for bleeds but not at the inferior turbinates (A). Although posterior epistaxis (originates from the posterior two thirds of the nasal cavity) can be quite severe and much more difficult to control neither the adenoids (C) or the superior turbinates (D) are the most common location for epitaxsis.

A 27-year-old man presents to your office with a complaint of numbness and weakness in his feet and fingers that began 24 hours ago. He tells you that two weeks ago he had an upper respiratory infection, but is otherwise healthy. Physical exam reveals right-sided facial droop and absent deep tendon reflexes. Which of the following is the most likely diagnosis? Botulism Guillain-Barré syndrome Lyme disease Multiple sclerosis

Correct Answer ( B ) Explanation: Guillain-Barré syndrome (GBS) is a post-infection, immune-mediated polyneuropathy. The cardinal feature of GBS is progressive, ascending, symmetric muscle weakness that begins in the lower extremities. Two-thirds of patients with GBS report a viral or bacterial infection prior to the onset of symptoms, most commonly upper respiratory or gastrointestinal. Muscle weakness progresses over hours to days and may eventually affect the muscles of respiration. Diagnosis is initially made based on clinical presentation and may be confirmed with cerebrospinal fluid analysis or neurophysiology studies including nerve conduction studies or needle electromyography. Patients diagnosed with GBS need to be hospitalized for close monitoring until it is determined where the patient is in the course of the disease. Many patients develop respiratory failure and require admission to the intensive care unit. Supportive care is the most important treatment modality for patients with GBS. Patients should receive deep vein thrombosis prophylaxis, bladder and bowel care, physical therapy and emotional support as part of their management plan. Prognosis ranges from full recovery to death, based on severity of symptoms and progression of the disease. Botulism (A) is a neurologic disorder that causes acute neuroparalysis. The clinical presentation of botulism is different from GBS due to patients with botulism having symmetric descending muscle paralysis or weakness instead of the ascending symptoms found in GBS. The second stage of Lyme disease (C) has neurologic manifestations such as facial palsy, headache and meningitis. This follows the first stage, which involves the classic skin rash, erythema migrans, and a flu-like febrile illness. Multiple sclerosis (D) presents with sensory and muscular symptoms that occur months or years apart and affect different locations in the body.

A 52-year-old man with a history of cirrhosis presents with worsening confusion. His wife reports he has been compliant with his lactulose at home. Which of the following is a common cause of worsening hepatic encephalopathy? Cardiac ischemia Constipation Hyperkalemia Pancreatitis

Correct Answer ( B ) Explanation: Hepatic encephalopathy is a state of confusion caused by acute and chronic liver disease. The pathophysiology involves many pathways but is directly related to the impaired liver's inability to perform its regular metabolic function. Ammonia is used as a marker of the process. Ammonia that is absorbed in the GI tract is metabolized to urea by the liver for excretion. When this does not occur, ammonia levels increase and ultimately cross the blood-brain barrier leading to cerebral dysfunction. Hepatic encephalopathy covers a spectrum of disease from mild cognitive impairment to coma. On physical examination, asterixis is characteristic of the process ("flapping" of the wrist when it is extended). There are many causes of acutely worsening hepatic encephalopathy including constipation due to an increase in intestinal ammonia production and subsequent absorption. Other causes include renal failure, gastrointestinal bleeding, infection, medications with central nervous system activity, and diuretic therapy leading to hypokalemia and alkalosis. Cardiac ischemia (A) is not a common cause of hepatic encephalopathy. It is, however, important to think of ischemia in the differential of patients in diabetic ketoacidosis. Hyperkalemia (C) is not typically linked with worsening encephalopathy but hypokalemia is. With decreased serum potassium and the alkalosis that may be associated with this, there is increased conversion of NH4+ to NH3 leading to increased serum levels of ammonia. Pancreatitis (D) alone is not an independent cause of hepatic encephalopathy but in severe cases may lead to dehydration and electrolyte or pH disturbances which can exacerbate the condition.

Which of the following types of hemoglobin does hydroxyurea increase? HbAA HbF HbSC HbSS

Correct Answer ( B ) Explanation: Hydroxyurea is the only drug approved by the U.S. Food and Drug Administration for treatment of sickle cell anemia. It works by increasing levels of HbF (fetal hemoglobin). HbF concentrations vary among patients with sickle cell anemia (homozygosity for HbS) and among erythrocytes of each individual. Because HbF inhibits HbS polymerization, its concentration within each cell and its distribution among all cells influence cell heterogeneity. In recent studies, use of hydroxyurea resulted in a reduction in cumulative mortality by almost 40%. Children have a more robust HbF response to hydroxyurea than adults. Cancer and leukemia have been reported in patients with sickle cell disease treated with hydroxyurea, but whether the incidence is higher than in the general population is not known. HbAA (normal individual) (A), HbSS (sickle cell disease) (D) and HbSC (hybrid S and C disease) (C) are not affected by hydroxyurea.

Which of the following tuberculosis medications is commonly associated with lupus-like syndrome? Ethambutol Isoniazid Levofloxacin Rifampin

Correct Answer ( B ) Explanation: Isoniazid is one of a number of drugs that is associated with the development of a lupus-like syndrome. Drug-induced lupus is SLE-like but self-resolving. The illness is characterized by arthralgias, rash, serositis and myalgias. It can be caused by a number of medications but the most common are hydralazine, isoniazid, procainamide, methyldopa, chlorpromazine, quinidine, minocycline and phenytoin. Patients with drug-induced lupus rarely have major organ involvement. Cessation of the offending medication leads to resolution. Ethambutol (A), levofloxacin (C) and rifampin (D) are drugs used in the treatment of tuberculosis but have not been linked to the development of drug-induced lupus.

A 35-year-old woman makes an appointment to establish primary care. Three months ago she was diagnosed with myasthenia gravis which is now only mildly symptomatic and is properly treated by a neurologist. Which of the following would you most expect to find during her intake physical examination? Hyporeflexia Ptosis Sensory abnormalities Weak hypothenar muscles

Correct Answer ( B ) Explanation: Myasthenia gravis (MG) is a disease of neuromuscular transmission characterized by fluctuating weakness and fatigability of certain muscles, namely those of the eyes, face and other cranial nerves. Pathological findings include a widened synaptic cleft, decreased surface area of the postsynaptic button, decreased number of postsynaptic acetylcholine receptors and the presence of anti-acetylcholine receptor antibodies. Symptoms typical of MG are drooping eyelids (ptosis), diplopia, strabismus, dysphonia, dysarthria and dysphagia. Electrodiagnostics are primarily used in confirming a diagnosis. Treatment includes anticholinesterase drugs (-stigmine), thymectomy, corticosteroids, plasmapheresis and intravenous immunoglobulin. Hyporeflexia (A), or areflexia, is not characteristic of MG. MG is a disorder of the motor, not sensory, nerves. Sensory exam (C) is intact in those with MG. Severe, advanced cases of MG may show muscle weakness, however, it is usually proximal, not distal. The hypothenar muscles (D) would not likely be affected in MG. Very few cases display distal weakness.

What is the most common cause of rectal bleeding in an adult? Colon cancer Diverticulitis Hemorrhoids Inflammatory bowel disease

Correct Answer ( C ) Explanation: Hemorrhoids are the most common cause of rectal bleeding in adults. Hemorrhoids are dilated veins and venules of the hemorrhoidal plexus. They are associated with constipation, straining with defecation, pregnancy, low-fiber diets, increased portal pressure and increased intrabdominal pressure. Internal hemorrhoids are proximal to the dentate line and are painless. External hemorrhoids are distal to the dentate line and are painful. Patients report bright red streaks on the toilet paper, on the outside of a firm stool or dripping into the toilet. External thrombosed hemorrhoids can be very painful and have a blue-purplish appearance. Treatment is nonsurgical in the majority of patients. Nonsurgical treatment consists of Sitz baths, topical analgesics, topical steroids, increasing fiber and water in the diet, not straining on the toilet, and laxatives. Surgical excision is utilized on acute thrombosis in extremely painful situations. Colon cancer (A) can present with rectal bleeding depending on the location of the tumor. Tumors located in the ascending colon are associated with occult bleeding usually due to ulceration. This can be detected by a fecal occult blood test or colonoscopy. Tumors of the transverse colon usually do not present with bleeding but with changes in the character of the stool. Tumors involving the descending colon can present with hematochezia and changes in bowel habits. Diverticulitis (B) is occasionally associated with rectal bleeding due to inflammation of the colon. It most commonly occurs in the descending or sigmoid colon. Inflammatory bowel disease (D) includes ulcerative colitis (UC) and Crohn's disease. Both disease can have symptoms of bloody diarrhea, abdominal cramping, mucous in the stool and other bowel changes. Crohn's can have systemic symptoms including uveitis and rheumatologic symptoms such as joint pain. However, it is not the most common cause of GI bleeding in adults.

A morbidly obese 32-year-old woman has a 4-month history of hypertension that has been nonresponsive to multiple antihypertensive medications. Blood pressure readings over the past month have an average diastolic pressure of 115 mm Hg. Her serum creatinine is 1.5 mg/dL. Which of the following diagnostic tests is most appropriate at ruling-in the most common cause of this refractory hypertension? Captopril renography Gadolinium-enhanced magnetic resonance angiography KUB radiograph Renal vein renin assays

Correct Answer ( B ) Explanation: Systemic hypertension is a common disease in America. Most cases are essential, however, secondary hypertension exists and is most commonly caused by renal vascular hypertension, with renal artery stenosis (RAS) being the most common pathology. Atherosclerosis and fibromuscular dysplasia are the most common underlying sources of renal artery stenosis. RAS accounts for 5% of all systemic hypertension. RAS is especially prevalent in cases of refractory, malignant or accelerated hypertension, as well as in significant diastolic hypertension in those less than 35-years-old. Clinical manifestations include abdominal or renal bruit, deteriorating renal function and other signs of generalized atherosclerosis. Diagnosis can be made with duplex Doppler ultrasonography, however, this modality may miss pathology of small arteries or in obese people. If clinical suspicion is high, and renal function is normal, the recommended initial diagnostic test is computed tomographic angiography or renal angiography. Consider captopril renography instead if the above tests are not desired or are contraindicated. If the clinical suspicion is high, and renal function is diminished, gadolinium-enhanced magnetic resonance angiography is the test of choice. Captopril renography (A) is not a reliable test in patients with poor renal function. A KUB radiograph (C) cannot identify renal artery stenosis. Invasive renal vein renin assays (D) should not be used as a screening test in those with suspected RAS and poor renal function.

A football player presents with acute anterior knee pain and significant knee effusion. He felt a "pop" while he was tackled. Which of the following physical examination tests is best used to diagnose a tear of the anterior cruciate ligament? Anterior drawer test Lachman test McMurray test Patellar grind test

Correct Answer ( B ) Explanation: The anterior cruciate ligament (ACL) is the primary stabilizer of the knee. It resists anterior translation of the tibia on the femur and is the most frequently injured major ligament of the knee. A tear results from deceleration, hyperextension or marked internal rotation of the tibia on the femur. The majority of ACL tears occur during sporting activities such as skiing, football, soccer, and basketball. Clinically, there is usually rapid development of hemarthroses, causing significant swelling. Almost half of individuals report feeling or hearing a pop, which is the most reliable factor. There is usually instability of the knee or a "wobbly" feeling. The Lachman test is the most sensitive test (approximately 98% sensitivity) showing increased anterior tibial displacement and a soft end point. MRI is rarely necessary to make a diagnosis. Rest, ice, crutches, immobilization and NSAIDs are common initial treatment options. Some competitive young patients may choose reconstruction as the best possible chance to return to successful play. The anterior drawer test (A) carries a 50% false-negative rate. As such, it is much less appropriate then the Lachman test for diagnosing ACL injury. The McMurray test (C) is used to diagnose meniscal, not ACL, injury. The patellar grind test (D) evaluates the articular surface of the patella, and is used in the evaluation of patellofemoral syndrome, not ACL tears.

A seven-year-old boy is in the clinic with his parents for evaluation of fatigue. He easily gets tired in the middle of class and has been sent home twice for the past week due to decreased energy. He has been doing well in school although he had a history of repeating second grade. He denies fever, abdominal pain, bone pain, or weight loss. On physical exam, height is at the 5th percentile, weight at the 50th percentile, hypopigmented spots on the chest, and pallor. Laboratory exam reveals macrocytic anemia, thrombocytopenia, and leukopenia. Which of the following is the most likely diagnosis? Diamond-Blackfan syndrome Fanconi anemia Immune thrombocytopenia Schwachman-Diamond syndrome

Correct Answer ( B ) Explanation: The boy in the vignette has signs, symptoms, and laboratory findings that are suspicious for Fanconi anemia (FA). FA is the most common form of inherited aplastic anemia. Most children with FA are diagnosed between six and nine years of age. FA is characterized by pancytopenia with macrocytic anemia and congenital malformations. Characteristic congenital malformations are present in up to 60 percent to 70 percent of affected children which include short stature, hypopigmented spots and café-au-lait spots, abnormality of thumbs, microcephaly or hydrocephaly, hypogonadism, and developmental delay. Patients with FA are at high risk for developing a malignancy, particularly myelodysplastic syndrome, acute myeloid leukemia, and squamous cell carcinoma of the head and neck or vulva. The hematologic findings in patients with FA evolve over months to years. Patients often present with mild to moderate thrombocytopenia. Mild leukopenia also may be seen as an initial presentation. Anemia is often macrocytic and frequently mild on presentation. About 70 percent of children with FA will have evidence of bone marrow failure by age 10. Diamond-Blackfan syndrome (A) presents with pure red cell aplasia and characteristic features that include hypertelorism, flat nasal bridge, ear anomalies, and high arched or cleft palate. Immune thrombocytopenia (C) presents with mucocutaneous bleeding with thrombocytopenia as the only abnormality detected on complete blood count. Schwachman-Diamond syndrome (D) usually presents in infancy with exocrine pancreatic dysfunction, bone marrow failure, and skeletal anomalies. Neutropenia is the most common manifestation of marrow failure.

A 12-year-old girl presents with a patch of hair loss with fine scaling. Occipital adenopathy is present on examination. What treatment is indicated? Ketoconazole shampoo Oral griseofulvin Topical corticosteroids Topical nystatin

Correct Answer ( B ) Explanation: The child's presentation is consistent with tinea capitis. Tinea capitis is dermatophytic scalp infection caused by Trichophyton or Microsporum species. Dermatophyte infections are common in both immunocompetent and immunocompromised individuals, but rare presentations should raise suspicion of immunodeficiency. The most common presentation of tinea capitis is an irregularly defined patch of scaly skin that enlarges and later causes alopecia. The infection is often not noticed until alopecia occurs. Affected hair may also be broken, resulting in curved, "comma," hairs or corkscrew-shaped hairs. Occipital lymphadenopathy is often present. The lesion may also progress rapidly to a kerion. Diagnosis is confirmed on potassium hydroxide (KOH) preparation of scalp scrapings. In contrast to tinea corporis, eradication of tinea capitis requires oral antibiotic therapy. Oral griseofulvin has been a mainstay of treatment for children with tinea capitis and has an excellent safety profile. It is administered for six to 12 weeks. Terbinafine, although less studied in children, is an alternative therapy. Recently, randomized trials of shorter courses of oral fluconazole and itraconazole have shown similar efficacy to griseofulvin for the treatment of tinea capitis. Ketoconazole shampoo (A) is used to treat seborrheic dermatitis of the scalp, but it is not sufficient to eradicate tinea capitis. Notably, oral ketoconazole is no longer recommended for the treatment of tinea capitis by the US FDA due to safety concerns. Topical corticosteroids (C) are contraindicated in dermatophyte infections, as they may exacerbate the infection. Additionally, steroids may alter the appearance of the infection, making later assessment and diagnosis more difficult. Topical steroids are used to treat alopecia areata, an autoimmune alopecia. The alopecia of alopecia areata has clearly defined borders and does not demonstrate broken hairs. In this child's case, the scaling of the lesion also makes tinea capitis more likely than alopecia areata. Topical nystatin (D) is recommended for the treatment of candidal skin infections, but nystatin is not effective in the treatment of dermatophytes.

A 32-year-old woman presents with an increase in vaginal secretions. You decide to perform a potassium hydroxide wet preparation of a sample. This test evaluates which of the following secretion qualities? Color Odor pH Viscosity

Correct Answer ( B ) Explanation: The main finding in vaginitis is increased vaginal discharge, a symptom which is 80-90% associated with a biologic organism and 10-20% associated with fluctuating chemicals or hormones. A common office test is the KOH "whiff" wet preparation test, in which a secretion sample is mixed with saline and 10-20% potassium hydroxide. The presence of a "fishy" amine odor represents a positive test, while the absence of this abnormal amine-like odor represents a negative result. Normal vaginal secretions, which are mainly comprised of cervical mucus, have a pH of 3.8 - 4.2, are clear or white and have a negative KOH test. Secretions become more basic (pH >4.5), thin, adherent, and gray in bacterial vaginosis and frothy-green in trichomoniasis. White, thick, "cottage-cheese like" secretions are associated with candida vaginitis. The KOH test is mostly positive in bacterial vaginosis and trichomoniasis, but negative in candidiasis. Color (A) is visually inspected, pH (C) is tested via an indicator strip and viscosity (D) is evaluated by rubbing the secretion between the fingers.

A 61-year-old woman presents to the Emergency Department after several episodes of syncope. Her rhythm strip is shown above. Which of the following atrioventricular blocks is present? First-degree AV block Second-degree Mobitz type I AV block Second-degree Mobitz type II AV block Third-degree AV block

Correct Answer ( B ) Explanation: The patient has a second-degree Mobitz type I AV block. A first-degree AV block refers to slowed conduction through the AV node, resulting in a prolonged PR interval, but preserved conduction of each sinus-generated impulse. In second-degree AV block, there is intermittent conduction through the AV node—some atrial impulses are conducted through to the ventricles while others are blocked. There are two types of second-degree block, Mobitz type I and type II. In second-degree Mobitz type I block, also referred to as Wenckebach, there is progressive delay in each AV conduction until AV conduction is finally blocked. On electrocardiogram, this appears as progressive lengthening of each PR interval until a QRS complex is eventually dropped and the cycle repeats. Grouped beating, or clustering of QRS complexes separated by a pause from the dropped beat, is characteristic of second degree type I AV block. Mobitz type I block is often transient, seen in association with inferior wall myocardial ischemia, medication toxicity, or cardiac surgery. Treatment is usually not necessary unless it is associated with a very slow rate with signs of hypoperfusion. In which case atropine, which increases the rate of sinus node firing and conduction through the AV node, is usually effective. The other type of second-degree AV block, Mobitz type II, is more ominous. Type II block is characterized by a constant PR interval with periodic non-conducted impulses, seen as absence of a QRS complex. Because Mobitz type II blocks are associated with progression to third-degree, or complete heart block, patients should have transcutaneous pacer pads placed in anticipation of possible need. In third-degree AV block, there is no conduction through the atrioventricular node, and an escape pacemaker is responsible for the ventricular rate. On electrocardiogram, there is no association of P waves with QRS complexes, and the ventricular rate is 40-60 beats/minute, depending on whether a junctional or ventricular escape pacemaker takes over. Pacing is usually needed in complete heart block. First-degree AV block (A) shows prolonged PR interval, but no dropped beats. Second-degree Mobitz type II AV block (C) is characterized by a constant PR interval with sporadic dropped beats. Third-degree AV block (D) is characterized by no association between P waves and QRS complexes.

A patient who is undergoing a transesophageal echocardiogram suddenly develops profound hypoxemia and cyanosis. His respiratory rate is 18 breaths/minute, but his saturation remains at 80% despite adequate application of supplemental oxygen by non-rebreather mask. A quick review of the medication record reveals that he received benzocaine, fentanyl, midazolam, and propofol during the procedure. What substance should you administer to treat his underlying condition? Flumazenil Methylene blue Naloxone Phenylephrine

Correct Answer ( B ) Explanation: This patient has methemoglobinemia. Methemoglobin is a substance that is present in the conversion pathway of iron to deoxyhemoglobin. It is always present in red blood cells, but it is constantly enzymatically metabolized under normal circumstances. Certain medications, including benzocaine, can disrupt this pathway, leading to an accumulation of methemoglobin and reducing the oxygen-carrying capacity of the blood. It may also alter the appearance of the arterial blood and make it appear chocolate colored. To confirm methemoglobinemia, an arterial blood gas with co-oximetry is needed to show the levels of methemoglobin and oxyhemoglobin. The classic finding in methemoglobinemia is a pulse oximetry reading around 80-85% despite administration of supplemental oxygen. In these patients, the PaO2 may be normal but their pulse ox is low because the pulse oximeter cannot detect methemoglobin Besides providing respiratory support, the treatment is methylene blue, which converts the methemoglobin to oxyhemoglobin. Fentanyl, midazolam, and propofol could all cause respiratory depression in this patient. Given the lack of improvement in saturation with application of supplemental oxygen and the normal respiratory rate, they are unlikely to be the cause. Flumazenil (A) is a benzodiazepine antagonist and naloxone (C) is an opiate antagonist. Phenylephrine (D) is a vasoconstrictor that is commonly used to treat acute hypotension, often caused by induction/sedation agents like propofol.

Which of the following patients should receive prophylactic antibiotics to prevent endocarditis? 18-year-old pregnant woman with a history of a repaired congenital heart defect, with an impending vaginal delivery 19-year-old woman with a history of endocarditis who is undergoing a dental extraction 20-year-old man with a prosthetic heart valve who requires a Foley catheter due to urinary obstruction 21-year-old man with a history of a heart transplant and valvulopathy who is undergoing suture repair of a facial laceration

Correct Answer ( B ) Explanation: This patient meets high-risk criteria (history of endocarditis) and requires antibiotic prophylaxis. The American Heart Association has published guidelines regarding prophylaxis for infective endocarditis in high-risk patients undergoing dental or invasive respiratory procedures. High-risk patients who are undergoing vaginal delivery (A), Foley catheterization (C), and suture repair (D) do not require antibiotic prophylaxis. In fact, prophylactic antibiotics are generally not needed with any genitourinary or gastrointestinal instrumentation and nearly all procedures that are performed in the ED.

A 12-year-old baseball player presents with knee pain. It occurs mainly during running the bases. Tenderness and edema are prominent about the right tibial tuberosity, and resisted knee extension is painful. The remainder of the knee examination is normal. Radiographic examination of this patient would most likely reveal which of the following abnormalities? Bipartite patella Heterotopic ossification Increased density of the femoral condyles Pars interarticularis defect

Correct Answer ( B ) Explanation: Tibial tuberosity osteochondritis is called Osgood-Schlatter disease (OSD). Repetitive quadricep contraction can lead to avulsions at the secondary ossification center between the tibial tuberosity and the patellar tendon. Incidence is greatest in adolescent boys who are active in sports. Symptoms include anterior proximal tibial pain, which is worse with running, jumping and prolonged sitting. Examination typically reveals unilateral tibial tuberosity tenderness and edema with restricted and tender kneeling. Lateral radiographs typically show spicules of heterotopic ossification anterior to the tibial tuberosity. Intermittent ice, NSAIDs and rest are common treatment options. Immobilization may be required for recalcitrant cases. Surgery is commonly unnecessary, and prognosis is typically good. Bipartite patella (A) may present with superolateral patellar pain, but not commonly with tibial tuberosity pain. Increased density of the femoral condyles (C) is not present in OSD. Increased density of the femoral head is common in Legg-Calve-Perthes disease, the childhood disorder of hip avascular osteonecrosis. A defect in the pars interarticularis (D), the bony junction of the superior and inferior articular processes of the vertebral posterior columns, can lead to spondylolisthesis and back pain, not knee pain.

A 37-year-old woman presents to the ED for epistaxis. After ten minutes of direct pressure, application of topical vasoconstrictors, inspection with a nasal speculum and suctioning, you note a significant amount of active bleeding from a vessel in Kiesselbach's plexus on the right side. What is the next best step in managing this patient's epistaxis? Cautery with silver nitrate Irrigation with normal saline Placement of anterior nasal packing Placement of posterior nasal packing

Correct Answer ( C ) Explanation: Management of epistaxis should always be handled in a stepwise fashion. First, the patient's airway, breathing, and circulation must be addressed. Once deemed stable, the examiner should attempt to determine if the bleeding is coming from an anterior (most commonly a bleeding vessel in Kiesselbach's plexus) or a posterior source (most commonly one or more branches of the sphenopalatine artery). In certain cases, it may be difficult to differentiate the two, and treatment for anterior epistaxis should be pursued first. If such treatment fails, then posterior epistaxis should be suspected. To manage anterior epistaxis refractory to direct pressure, the nares should be inspected and suctioned. Topical vasoconstrictors and anesthetic may add to patient comfort and make visualization easier. Preformed nasal packing products or commercially available balloon catheter devices are convenient alternatives to traditional anterior nasal packing and are now used by most clinicians. Anterior packs are usually left in for two to five days. Premature removal may result in re-bleeding. Although the necessity of antibiotics for short-term nasal packing is unproven, most clinicians prophylactically start patients on antibiotics with gram-positive coverage. Cautery with silver nitrate (A) may be attempted but generally will not work with an actively bleeding vessel. Irrigation (B) does not have a role in anterior epistaxis and may compromise a patient's airway. Bleeding from Kiesselbach's plexus is indicative of an anterior source. Posterior epistaxis is uncommon yet life-threatening. Treatment is achieved by deep nasal packing, extending to the posterior nasopharynx (D). All patients with a posterior source of bleeding should be admitted to the hospital and have a consultation with an otolaryngologist.

Which of the following bone fractures is most commonly involved in acute compartment syndrome? Fibula Radius Tibia Ulna

Correct Answer ( C ) Explanation: Muscles in the human body are divided into compartments lined with fascial membranes. Acute compartment syndrome (ACS) occurs when increased tissue pressure within a compartment results in nerve and muscle ischemia. ACS generally develops after a significant trauma such as a fracture, but may also occur with minor trauma or due to a nontraumatic etiology. The most common sites for ACS to occur are the leg and arm. Long bone fractures are the cause of 75% of cases of ACS, with the tibia being the most common bone involved. ACS is considered to be a surgical emergency. Measurement of compartment pressures assists in determination of the need for fasciotomy, which is the definitive treatment for ACS. Frequent, serial examinations are critical in the management of patients at risk for ACS. Prognosis is good in patients with early diagnosis and appropriate intervention. Isolated fibula (A) fractures are uncommon. When a fibula fracture occurs, it generally occurs as the result of a tibial fracture. The bones of the forearm such as the radius (B) and ulna (D) are the second most common types of fracture involved in ACS.

Which of the following disorders is characterized by intermittent color change of extremities? Acrocyanosis Buerger's disease Raynaud's disease Reactive arthritis

Correct Answer ( C ) Explanation: Raynaud's disease is a vasospastic disorder characterized by triphasic color change. Raynaud's disease is defined as vasospasm in small arteries and is characterized by intermittent triphasic color changes: pallor, cyanosis, and then rubor. It is the most common vasospastic disorder and affects women more than men (5:1). The main criteria for diagnosis are: 1) episodes precipitated by cold, 2) bilateral symptoms, 3) no cause of secondary Raynaud's phenomenon, 4) symptoms occurring for 2 or more years and 5) gangrene is absent. Patient's experience attacks lasting 15-60 minutes. Pallor results in a chalk white appearance of the affected digits. Rewarming leads to restoration of normal color and sensation. Acrocyanosis (A) is an uncommon vasospastic disorder characterized by persistent, symmetrical cyanosis of the fingers and hands. It is painless. Buerger's disease (B), or thromboangiitis obliterans, is an inflammatory occlusive disease of medium and small arteries of the hands and feet. Reactive arthritis (D) (formerly known as Reiter's syndrome) is characterized by arthritis, urethritis and uveitis.

Which of the following is correct regarding cardiac markers in regards to myocardial infarction? A Creatinine phosphokinase (CK) is specific to cardiac tissue B Myocardial infarction can be ruled out with a single serum myoglobin C Serum troponin is more sensitive than creatinine phosphokinase (CK-MB)Correct Answer D Troponin levels return to normal in 2-3 days

Correct Answer ( C ) Explanation: Serum troponin is a more sensitive marker of myocardial infarction than creatinine phosphokinase (CK-MB). Biochemical markers (troponin, CK-MB, myoglobin) are essential in the diagnosis and risk stratification of myocardial infarction. Troponin I and T are myocardium specific proteins that are released from myocardial cells after cell damage occurs. Both have been found to be highly specific and sensitive for the early detection of myocardial injury and have supplanted the use of other biomarkers. The presence of serially negative troponins also predicts a low risk for an event. Troponin levels may not be detectable for up to 12 hours after symptoms onset in some patients with acute myocardial infarction. However, with highly sensitive troponin assays, detection may be possible as early as 3 hours. CK (A) is a protein that is found in large quantities in cardiac and skeletal muscle and is thus nonspecific. Myoglobin (B) is found in all muscle tissue and rises earlier in myocardial infarction than troponin. However, a single normal level does not rule out a myocardial infarction. While CK levels return to normal within 48-72 hours, troponin levels take up to 5-14 days (D) to return to baseline.

A patient presents with vomiting, rapid breathing, and confusion after a week of not using his prescribed medications. You order serum electrolytes, serum creatinine and urea nitrogen, arterial blood gases, lactate levels, and a toxicology screen. You calculate an anion gap to be >18 mEq/L. Which of the following tests is most likely to be useful in evaluating the etiology of suspected metabolic acidosis? Aldosterone Amylase Ketones Urinary pH

Correct Answer ( C ) Explanation: The initial work-up of metabolic acidosis is determining if there is an increased serum anion gap. Anion gap (AG) is the difference between the sodium, chloride, and bicarbonate concentrations (Na - [Cl + HCO3]). A normal AG is 3-11 mEq/L, whereas an increased anion gap is >11-20 mEq/L. If an increased anion gap is found, check for ketones, lactate, BUN and creatinine levels, and a toxicology screen. A common mnemonic for anion gap metabolic acidosis etiologies is MUDPILES: Methanol, Uremia/renal failure, Diabetic ketoacidosis, Paraldehyde (component of anticonvulsants, hypnotics and sedatives)/propylene glycol, Isoniazid/iron, Lactic acidosis (common in cardiopulmonary failure, sepsis, alcoholism, ischemic bowel disease, short bowel syndrome, carbon monoxide, and cyanide poisoning and cancer), Ethylene glycol/ethanol, and Salicylates. If there is no increase in the anion gap, consider the etiologies of non-anion gap metabolic acidosis: diarrhea, intestinal drainage or fistula, renal tubular acidosis, rapid infusion of bicarbonate-free fluids, ureteral diversions, and ingestion of toluene, cholestyramine, sevelamer or acetazolamide. Further work-up of non-anion gap metabolic acidosis includes urinary anion gap, urine pH, serum potassium and the fractional excretion of bicarbonate. Aldosterone (A) levels are used in the evaluation of metabolic alkalosis, as hyperaldosteronism drives the kidneys to waste hydrogen ions and retain bicarbonate ions. Amylase (B) is used in the evaluation of pancreatitis, other pancreatic disease and salivary gland diseases. Urinary pH (D) is more important in the evaluation of non-anion gap metabolic acidosis. It is used to differentiate the types of renal tubular acidoses.

Which of the following represents the classic order of the stages of grief? Anger, Denial, Bargaining, Depression, Acceptance Bargaining, Anger, Denial, Depression, Acceptance Denial, Anger, Bargaining, Depression, Acceptance Depression, Denial, Anger, Bargaining, Acceptance

Correct Answer ( C ) Explanation: The stages of mourning and grief represent a universal experience across cultures and backgrounds. The five stages of normal grief were first proposed by Elisabeth Kubler-Ross in 1969. People spend different amounts of time in each stage of grief. The first reaction to bad news is denial of the reality of the situation. Denial is a defense mechanism that buffers the immediate pain of shock. This is followed by anger that may be aimed at people or inanimate objects. It can often be directed at the doctor who delivered the news. Bargaining is the next stage and is a response to feelings of helplessness and vulnerability. Depression typically follows bargaining. There is often depression over the practical implications of the news as well as a deeper emotional depression. Finally, acceptance is reached. Although the stages of grief can be experienced in different orders, the classic teaching is that Denial precedes anger (A), bargaining (B) and depression (D).

A 23-year-old man presents with redness and discharge from his right eye. Visual acuity is normal. Which of the following treatments should most likely be avoided in this patient? Ophthalmic antibiotic drops Ophthalmic antibiotic ointment Topical corticosteroids Warm compresses

Correct Answer ( C ) Explanation: This patient presents with conjunctivitis and should be treated with either ophthalmic antibiotic drops or ointment and warm compresses. Conjunctivitis is an inflammation of the bulbar and palpebral conjunctiva. It can be caused by trauma, toxic agents, allergies or viral and bacterial pathogens. Common bacterial agents include Streptococcus pneumoniae, Haemophilus influenzae, Staphylococcus organisms, Moraxella catarrhalis, and Neisseria gonorrhoeae. Although viral infections are the most common cause of conjunctivitis, it is often difficult to differentiate viral from bacterial causes leading many clinicians to treat all presentations as bacterial conjunctivitis. Patients typically present with eye redness, lid swelling, crusting of the eye in the morning and drainage. Pain, loss of vision and photophobia should not be present. Ophthalmic ointments may provide a soothing affect but can interfere with vision. Drops do not interfere with vision. Corticosteroids and eye patching should be avoided as these may exacerbate the infection. The standard treatment for acute conjunctivitis includes ophthalmic antibiotic drops (A) or ointment (B) and warm compresses (D).

A 30-year-old woman misses work and presents to the Emergency Department with severe pelvic pain rated at 6/10. She states it began yesterday with the onset of menstruation. She has regular cycles with normal blood flow amount, but has not had this pain before. She denies spine, urologic and rectal symptoms. Vital signs are normal, and physical examination as well as pelvic ultrasound is unremarkable. Serum beta-hCG is negative. Other than referral to a gynecologist for further evaluation, which of the following is the most appropriate initial treatment? Intravenous morphine Oral contraceptive pills Oral ibuprofen Oral pregabalin

Correct Answer ( C ) Explanation: This patient's working diagnosis is most concordant with primary dysmenorrhea, in which there is significant pain associated with the first few days of menses which alters normal activity or requires pain medication to control. Although secondary causes such as endometriosis and pelvic inflammatory disease are the most common misdiagnosis of primary dysmenorrhea, her stable presentation in this specialized setting, in conjunction with a normal initial diagnostic test, favors symptomatic control and follow-up evaluation. To treat dysmenorrhea, non-steroidal antiinflammatory medications (NSAIDs) and acetaminophen are helpful. Opioid analgesics (A) are considered second line if NSAIDs and acetaminophen fail to relieve the pain. Consider parenteral pain control if oral medications fail. Abdominopelvic heat is also beneficial. Contraceptive pills (B) are reserved for women who do not respond to pain medications. Pregabalin (D), a neuropathic pain medication, would not be appropriate in the acute setting, nor would it relieve the nociceptive pain of primary dysmenorrhea.

A 55-year-old man is being evaluated for sudden onset of chest pain. He describes the pain as sharp that is improved by sitting up and leaning forward. The chest pain is made worse by inspiration or coughing. An ECG shows new widespread ST segment elevation. Auscultation over the left sternal border is heard here. Which of the following is the most likely diagnosis? Aortic stenosis Mitral stenosis Patent ductus arteriosus Pericarditis

Correct Answer ( D ) Explanation: A pericardial friction rub is most commonly associated with pericarditis. It resembles the sound of squeaky leather and is often described as grating, scratching, or rasping. The pericardium is a double-walled sac around the heart. The inner (visceral) and outer (parietal) layers are normally lubricated by a small amount of pericardial fluid, but when inflammation of pericardium is present, the 2 walls move against each other with audible friction (the rub). In children, rheumatic fever is often the cause of pericardial friction rub. The friction rub is usually best heard between the apex and sternum but may be audible across the precordium. The murmur associated with aortic stenosis (A) is associated with an easily heard systolic, crescendo murmur loudest along the upper right sternal border at the 2nd right intercostal space radiating to the carotid arteries bilaterally. The murmur associated with mitral stenosis (B) is mid-diastolic and has a rumbling character. It is best heard with the bell of the stethoscope in the left ventricular impulse area with the patient in the lateral decubitus position. It usually starts with an opening snap. A patent ductus arteriosus (C) is classically associated with a machinery, crescendo/decrescendo murmur continuous from systole to diastole.

Which of the following sutures is absorbable? Nylon (Polyamide) Prolene (Polypropylene) Silk (Silk) Vicryl (Polyglactin)

Correct Answer ( D ) Explanation: Absorbable sutures are degraded and lose strength in less than 60 days. Absorbable sutures are generally used for subcutaneous and mucosal closures. Their highly reactive nature allows them to be broken down and absorbed over weeks. Vicryl is an absorbable braided polymer of lactide and glycolide with low reactivity and good strength making it suitable for subcutaneous and mucous membrane use. Prolene (B), Nylon (A), and Silk (C) are nonabsorbable.

You are treating a patient for primary amenorrhea. Her diagnostic evaluation has determined that the etiology is primary ovarian failure. Which of the following disorders is this patient at risk for? Cluster headaches Gastrointestinal bleeding Menorrhagia Osteoporosis

Correct Answer ( D ) Explanation: Amenorrhea can have lasting complications. It is imperative to thoroughly evaluate and treat women with amenorrhea to decrease the risks of abnormal sexual development, abnormal psychosocial development and infertility. One of the causes of amenorrhea is primary ovarian failure, a disease classification that encompasses several conditions which cause failure of ovarian structure or function, the most concerning aspect of which is estrogen deficiency. A hypoestrogenic patient can develop severe osteoporosis, leading to increased mortality due to femoral neck fractures. Hormone replacement therapy, with estrogen-progesterone combinations, helps to maintain normal bone density. Overall, the prognosis for patients with amenorrhea is good, provided that neoplastic sources have been excluded and hormone replacement therapy is considered. Migraine, and not cluster (A), are more prevalent in hypoestrogenic women. Estrogen deficiency does not lead to an increase in gastrointestinal bleeding (B). Menorrhagia (C), or heavy and/or prolonged menstrual flow, would not occur in someone with amenorrhea.

Which of the following is true regarding the characteristic rash of chickenpox? Lesions appear over 2-4 weeks with multiple stages present at once Lesions appear over 2-4 weeks with one stage present at a time Lesions appear over days and fade by the third day Lesions appear over days with multiple stages present at once

Correct Answer ( D ) Explanation: Chickenpox is a highly contagious but generally benign and self-limited viral disease caused by the varicella-zoster virus (also known as human herpesvirus 3). The disease is characterized by the sudden onset of fever, malaise, and a pustular maculopapular rash that can occur anywhere on the skin or mucus membranes. The lesions then become vesiculated followed by scabbing over the course of 3-4 days before resolving. Skin lesions appear in crops with multiple lesions of various stages appearing on the skin at the same time. Uncomplicated infection is generally treated with supportive measures, including antipyretic, antipruritic, and pain control medications. Antivirals such as acyclovir, valacyclovir, and foscarnet may also be initiated in severe disease or immunosuppressed individuals. Parents should be cautioned to avoid giving their children aspirin or aspirin-containing medications due to the risk of developing Reye's syndrome. The lesions of chickenpox appear suddenly rather than gradually (A). Smallpox lesions may appear similar to chickenpox lesions, however, they are found in the same stage (B) of development. Rubella (German measles) is associated with the sudden onset of a maculopapular rash that first appears on the face then rapidly spreads inferiorly to the neck, trunk, and extremities and fades by the third day (C).

A 16-year-old boy is taken to his doctor for snoring. His mother reports that his snoring keeps others in the house awake and that sometimes his breathing pauses during sleep with gasping or choking. His teachers report that he falls asleep frequently at school. On exam, he has a body mass index of 31 kg/m² and has enlarged tonsils. His symptoms are concerning for obstructive sleep apnea so the pediatrician refers him for an overnight polysomnography. Which of the following can be a longterm complication of obstructive sleep apnea? Cerebrovascular accident Lung scarring Nasal polyps Pulmonary hypertension

Correct Answer ( D ) Explanation: Children with obstructive sleep apnea (OSA) often are obese, have enlarged tonsils or have significant allergies. OSA can lead to abnormal growth and development, bedwetting, behavioral and learning problems, daytime sleepiness, and hyperactivity. Long-standing repetitive oxygen desaturations and hypercapnia episodes during sleep can lead to vascular remodeling and pulmonary hypertension which can lead to cor pulmonale (right ventricular hypertrophy). Treatment may include weight loss, managing allergic rhinitis, and removal of the adenoids and tonsils. Nonsurgical approaches to treatment include weight loss, CPAP, and bite guards that bring the lower jaw forward. Surgical treatments include uvulopalatopharyngoplasty, jaw surgery, or removal of the tonsils and adenoids. OSA does not lead to increased incidence of cerebrovascular accidents (A) but rather some of the same risk factors for OSA, such as obesity, also increase the risk of cerebrovascular accidents. Lung scarring (B) can be a long-term lung remodeling effect of asthma. Diseases associated with nasal polyp formation (C) include chronic rhinosinusitis, cystic fibrosis and Kartagener's syndrome.

A 4-year-old patient is brought to the emergency department by his parents after a choking incident that occurred one hour ago. The parents tell you that their son was eating steak, started to laugh and then began choking. Since the incident, he has been coughing and wheezing. Physical exam reveals unilateral diminished breath sounds with auscultation of the lungs. Which of the following is the most appropriate next step in management? Begin a course of antibiotics and steroids Chest X-ray Flexible bronchoscopy Rigid bronchoscopy

Correct Answer ( D ) Explanation: Foreign body aspiration (FBA) can be a life-threatening event and is a common cause of morbidity and mortality in children. The majority of pediatric FBA incidents occur in children less than 3 years of age. Types of foreign bodies that are commonly aspirated by children include peanuts, popcorn, other nuts and seeds, food particles and pieces of toys. FBA is a true medical emergency if there is a complete airway obstruction. Children with cyanosis, altered mental status and severe respiratory distress require immediate medical intervention including life support and rigid bronchoscopy to remove the foreign body. All cases of suspected FBA require that the tracheobronchial tree be examined. Rigid bronchoscopy is used for this examination and is both a diagnostic and management tool for partial and complete obstructions caused by FBA. A course of antibiotics and steroids (A) is indicated in a case of FBA diagnosed days or weeks after the aspiration occurs. The foreign body can cause inflammation and infection in the airway and patients present with signs and symptoms of pneumonia. Chest X-ray (B) may detect FBA if the aspirated object is radioopaque or depending on the severity of the obstruction. Normal findings on a chest X-ray do not rule out FBA since foods are not radioopaque and are commonly aspirated by pediatric patients. Flexible bronchoscopy (C) is often used in children with pneumonia or other respiratory symptoms. Use of flexible bronchoscopy for diagnosis of FBA may dislodge the object and cause a partial obstruction to become complete.

Which of the following medications is the preferred outpatient treatment of hypertension in pregnancy? Hydrochlorothiazide Lisinopril Losartan Methyldopa

Correct Answer ( D ) Explanation: Hypertension is defined as a systolic blood pressure ≥ 140 mm Hg or diastolic blood pressure ≥ 90 mm Hg measured on at least two separate occasions. Hypertension in pregnancy is classified as chronic (present prior to pregnancy) or gestational (begins after the first 20 weeks of pregnancy). Hypertension in the first 20 weeks of pregnancy is attributed to preexisting hypertension. Pregnant women with hypertension are at increased risk of adverse pregnancy outcomes, such as preeclampsia, abruptio placenta, preterm birth, and low-birthweight infants. Studies fail to demonstrate that antihypertensive therapy in pregnancy reduces these complications, and thus treatment of hypertension is indicated for maternal benefit. Mild uncomplicated hypertension in pregnancy does not require treatment. Treatment is indicated for severe hypertension (systolic blood pressure ≥ 160 mm Hg or diastolic blood pressure ≥ 105-110 mm Hg) or in the presence of end-organ damage (i.e.nephropathy or left ventricular hypertrophy). Caution must be used when selecting an antihypertensive agent since all antihypertensive drugs cross the placenta. Methyldopa is the most studied drug and has proven safety and efficacy in pregnancy. Methyldopa is started at a dose of 250 mg PO every 6 hours and titrated to effect. Labetalol is another recommended agent, and is started at 100 mg PO twice a day. Hydrochlorothiazide (A), and diuretics in general, are considered second-line agents. Angiotensin-converting enzyme inhibitors, such as lisinopril (B) and angiotensin receptor blockers, such as losartan (C) are contraindicated in pregnancy because of toxic effects on the fetus.

A 60-year-old man with a history of hypertension and paroxysmal atrial fibrillation is scheduled to undergo an elective dental extraction. His paroxysmal atrial fibrillation is rate controlled with a beta blocker and he is on chronic anticoagulation with warfarin. Anticoagulation must be discontinued for the procedure. His medications include metoprolol tartrate, lisinopril and warfarin. In addition to discontinuing warfarin, which of the following is the most appropriate next step in management? Bridge with aspirin Bridge with intravenous unfractionated heparin Bridge with low molecular weight heparin No bridging agent is necessary

Correct Answer ( D ) Explanation: No bridging agent is necessary. Interruption of oral anticoagulation in patients with atrial fibrillation is sometimes necessary for invasive procedures, and it must be determined whether bridging is indicated. If the patient has a low short-term risk as determined by a CHA2DS2-VASc score of 0-1 and the duration of interruption is less than 1 week, then bridging is not needed. This patient has a CHA2DS2-VASc score of 1 due to his history of hypertension; therefore, no periprocedural bridging is necessary. In this scenario it would be recommended to simply discontinue warfarin approximately 5 days before the procedure with no bridging agent. Warfarin can often be resumed 12-24 hours after the procedure if there is no active bleeding. If the patient has a high short-term risk with a CHA2DS2-VASc score of > 2, recent stroke, mechanical or rheumatic mitral valve or if the interruption is more than 1 week, then use of a bridging agent should be considered more strongly. In this setting, it would be appropriate to bridge with intravenous unfractionated heparin (B) or low molecular weight heparin (C). For atrial fibrillation patients with moderate risk features with a CHADS2 score of 3-4, a history of remote transient ischemic attack or stroke, or a mechanical aortic valve, management is individualized according to risks and benefits and is patient and provider dependent. It would not be appropriate to bridge with aspirin (A) as aspirin has no role in bridging.

A 44-year-old previously healthy man presents to the Emergency Department with low back pain that started two days ago. He recently moved to a new apartment and has been lifting boxes and furniture. He denies any bowel or bladder incontinence. On examination, he is afebrile. He has tenderness with palpation of the right lumbar paraspinal muscle which radiates to the buttock. Reflexes and strength are intact. Pain is exacerbated with movement. What is the next best step in management? Five day course of oral corticosteroids Magnetic resonance imaging of the lumbosacral spine Plain radiographs of the lumbosacral spine Prescribe ibuprofen and discharge home

Correct Answer ( D ) Explanation: Patients with uncomplicated musculoskeletal low back pain present with pain of the lumbar paraspinal muscles which can radiate to the buttock or posterior thigh. Pain is worse with movement of the trunk, but improves with rest. Strength, sensation, and reflexes will be intact and there will be no history of bowel or bladder incontinence. Presence of any of these neurologic findings on examination would be considered a red flag and should prompt further workup with imaging. Management of uncomplicated low back pain includes oral nonsteroidal anti-inflammatory medications (e.g. ibuprofen) and discharge home with primary care follow-up. Oral narcotic medications can be used in the short-term for breakthrough pain, but have increased side effects and are not considered first line treatment. Strict bed rest is no longer recommended as patients who remain active have an earlier resolution of symptoms. A five day course of oral corticosteroids (A) has not shown to be significantly beneficial in treatment of uncomplicated back pain. Given this patient has no concerning findings on history or physical examination, MRI of the lumbosacral spine (B) and plain radiographs of the lumbosacral spine (C) are not indicated.

A 19-year-old college student presents dead-on-arrival to the ED. A post mortem autopsy reveals significant calcification of the aortic valve cusps. Upon review of his medical history, you would most likely find documentation of a murmur heard in which of the following locations? Apex Left, fourth intercostal space Right, fourth intercostal space Right, second intercostal space

Correct Answer ( D ) Explanation: Proper cardiac auscultation begins with an understanding of which chest wall location is associated with which valve-sound. The aortic valve is best appreciated in the right, second intercostal space just lateral to the sternum. The pulmonic valve is best heard in the left, second intercostal space just lateral to the sternum. The tricuspid valve can be appreciated in the left, fourth intercostal space over the left sternal border. The mitral valve is best appreciated in the left, fifth intercostal space about the midclavicular line, also known as the apex. The murmur of aortic stenosis occurs during systole between S1 and S2. As such, it is also called an ejection murmur. It is best heard in the right, second intercostal space. Other characteristics include medium pitch, crescendo-decrescendo tonality and possible associated thrill. Aortic stenosis is due to calcification of the valve cusps. It is a common cause of sudden death, especially in children and adolescents. It can also be associated with rheumatic heart disease, atherosclerosis and congenital bicuspid valve malformation. The mitral valve is best heard at the apex (A). An abnormality of the tricuspid valve would be best heard in the left, fourth intercostal space (B). The right, fourth intercostal space (C) does not have a specific valve-sound associated with it.

A 6-week-old boy presents with a 3-week history of progressive non-bilious vomiting. There is no history of recent fever, diarrhea or blood in his stool. The physical exam reveals minimal weight gain over the past two weeks. Which of the following is the most likely diagnosis? Intussusception Jejunal atresia Midgut volvulus Pyloric stenosis

Correct Answer ( D ) Explanation: Pyloric stenosis is more common in boys however; a maternal history of pyloric stenosis increases the risk more than a paternal history. It is characterized by nonbilious vomiting, which is the initial symptom. The vomiting may or may not be projectile initially but is usually progressive, occurring immediately after feeding. The vomiting usually starts after 3 weeks of age, but symptoms can develop as early as the first week of life, and as late as the fifth month. After vomiting, the infant is hungry and wants to feed again. As vomiting continues, a progressive loss of fluid, hydrogen ion, and chloride leads to hypochloremic metabolic alkalosis with hypokalemia. Hyperbilirubinemia is also observed, most often unconjugated. On exam, a mass may be palpated above and to the right of the umbilicus in the mid-epigatrium beneath the liver's edge. The "olive mass" is best palpated after an episode of vomiting. A peristaltic wave may also be seen across the patient's abdomen after feeding. The diagnosis is most often made by ultrasound and treatment is surgical with a pyloromyotomy. Intussusception (A) occurs when a portion of the bowel telescopes into an adjacent segment. Ileocolic is the most common portion. This intestinal obstruction is most common between 3 months and 6 years of age. Most are idiopathic. Symptoms include severe colicky pain that tends to recur frequently with legs or knees flexed and straining efforts along with crying during the episode. Subsequently, lethargy is often noted. Vomiting is present early-on and infants may pass stool containing blood and mucus (currant jelly stool). An abdominal mass may be palpated in the RUQ. Intrauterine vascular accidents are generally thought to be the cause of jejunal atresia (B) where infarcted tissue is reabsorbed in the fetal intestine. Abdominal distension typically occurs within 12-24 hours of life after initiation of feedings. Vomiting also occurs and is often bilious. No masses are typically felt. Midgut volvulus (C) should be suspected in any infant with bilious vomiting. Most symptomatic intestinal malrotations are found in the first month of life. Initial symptoms are subtle and may be limited to irritability and progressive bilious vomiting. Late symptoms are abdominal distension and peritonitis. Guaiac-positive stool is usually present. No mass is usually felt on exam.

A 60-year-old African-American man with a history of hypertension and diabetes mellitus type II presents to your office for his annual exam. He admits to non-compliance with his routine medications and has been taking ibuprofen daily for the past six months due to headaches. Which of the following is the most useful initial imaging study to evaluate this patient for chronic kidney disease? CT scan with contrast CT scan without contrast MRI Renal ultrasound

Correct Answer ( D ) Explanation: Risk factors for chronic kidney disease include a history of hypertension, African ancestry, diabetes mellitus, and the use of NSAIDs. Ultrasound is the most useful imaging study to confirm that both kidneys are present, symmetric, and to estimate their size. It can also determine if a renal mass or obstruction is present. Bilateral small-sized kidneys indicate a diagnosis of chronic kidney disease that is long-standing. Kidneys may be of normal size with either acute or subacute renal disease. Evaluation of serum creatinine is the most important initial step in evaluating a patient with abnormal kidney function. Comparison of current and past measurements of serum creatinine can help determine if renal dysfunction is acute and therefore reversible. Serum creatinine concentration is used with age, sex, race and body weight to determine the estimated glomerular filtration rate. CT scan without contrast (B) and MRI (C) are used to diagnose renovascular disease. CT scan with contrast (A) should be avoided in patients who are suspected to have chronic kidney disease because radiographic contrast dye can induce renal failure.

A mother brings her 12-month-old daughter for routine vaccinations. She is up to date with her vaccinations. The girl has a history of a simple febrile seizure at 10 months of age and has a history of respiratory distress after eating eggs. She lives with her parents and her paternal grandmother who is currently undergoing chemotherapy for breast cancer. Because of anxiety of her daughter getting a fever and a febrile seizure, the patient's mother would like to spread out her vaccines. Today she would like her daughter to get the varicella vaccine. She would like to bring the patient back to the office in 2 weeks to get her measles-mumps-rubella (MMR) vaccine but you explain this is contraindicated. Why can the patient not get the MMR vaccine then? Her risk of vaccine-induced seizure She has an anaphylactic egg allergy She is living with an immunosuppressed family member She must wait 4 weeks between live vaccines

Correct Answer ( D ) Explanation: She must wait at least 4 weeks between getting live vaccines if they are not initially given concurrently. Live vaccines include the varicella vaccine, the MMR vaccine, the intranasal influenza vaccine, the rotavirus vaccine, the yellow fever vaccine, the BCG vaccine, the typhoid vaccine, the rabies vaccine, and the oral polio vaccine. Live vaccines must replicate to cause an immune response. Antibodies developed after the first live vaccine can interfere with viral replication of the subsequent vaccine if live vaccines are given apart but too close together. MMR and varicella vaccines are not contraindicated in children with severe egg allergies because they are derived from tissue culture (B), however, inactivated and live attenuated influenza vaccines are contraindicated in patients with severe egg allergies. Children with a mild egg allergy can receive the inactivated influenza vaccine and be observeed in the office for 30 minutes. Anaphylaxis to neomycin or another component of the MMR vaccine is a contraindication. Live vaccinations are not contraindicated for patients who live with immunocompromised individuals (C) but are contraindicated in many patients who are immunocompromised. However, MMR and varicella vaccines are recommended in patients with HIV who do not have severe immunosuppression. A history of febrile seizures (A) is not a contraidication for getting vaccinated. There is a risk of about 1 in 2,000 of a febrile seizure within 7 to 10 days of getting the MMR and varicella vaccines. The risk for seizures doubles for MMR-V compared to the administration of the MMR and varicella vaccines in children 12-23 months.

A 61-year-old man presents to the clinic for urinary urgency and frequency, nocturia, and dribbling urine consistently after he voids. He endorses a weak stream and a feeling that his bladder won't empty fully. You plan to start treatment for benign prostatic hyperplasia if his physical exam confirms it. Which of the following physical exam findings is most indicative of benign prostatic hyperplasia? A warm, boggy, prostate gland that is tender on palpation Asymmetric enlargement of the prostate gland with unilateral induration Normal size and texture of the prostate gland Smooth, firm, elastic enlargement of the prostate gland

Correct Answer ( D ) Explanation: Smooth, firm enlargement of the prostate gland is characteristic of benign prostatic hyperplasia (BPH). BPH is the most common benign tumor in men, with the incidence estimated at 50% of men by 60 years. Between ages 55-75, the likelihood of developing obstructive voiding symptoms rises significantly. Symptoms include hesitancy and straining to urinate, decreased force and caliber of the urine stream, and the need to urinate twice within 2 hours due to incomplete emptying. Post void dribbling can occur due to the residual urine in the bladder after voiding. Men may also develop irritative bladder symptoms including urgency, frequency, and nocturia. Though prostate size does not correlate with symptoms, a rectal exam will usually show some degree of smooth, firm enlargement. Induration should not be present, as this would be more concerning for malignancy. A distended bladder may be palpated. The decision to get a serum prostate-specific antigen level is controversial, as it is difficult to use this value to determine whether a patient has BPH as opposed to cancer. The American Urological Association symptoms index should be used to gauge severity of BPH and guide treatment. Recurrent urinary tract infections, gross hematuria, bladder stones, or kidney disease merit surgical resection and require referral to urology. Otherwise, mild symptoms can be managed with observation. Moderate to severe symptoms merit medical therapy, for which many options exist. Alpha-blockers will reduce symptoms by improving urine flow at the bladder neck. The 5-alpha reductase inhibitors, interrupts the processing of testosterone and can actually decrease the size of the prostate and improve symptoms. Six months are required to see improvement. Phosphodiesterase-5 inhibitors, like tadalafil, can offer improvement as well. Combination therapy is usually beneficial and tolerated. Asymmetric enlargement of the prostate gland with unilateral induration (B) is concerning for prostate cancer. This finding should alert the clinician to the need for further work-up, including a serum prostate-specific antigen, transrectal ultrasound, and biopsy. A warm, boggy prostate gland that is tender on palpation (A) is a typical finding in acute or chronic bacterial prostatitis. In the acute setting, this requires a urine culture with antibiotics guided by culture findings. A normal size and texture of the prostate gland (C) may occasionally be present in patients with underlying BPH. However, a normal prostate exam should alert the clinician to explore other causes of the obstructive and irritative voiding symptoms, such as urinary tract infections, urethral strictures, or neurogenic bladder.

A 15-year-old boy is brought by his father to your clinic for evaluation of a limp. This is accompanied by right hip and right knee pain, which he has noticed for about two weeks after he slipped while trying to sit on a chair. He denies fever and does not participate in sports. On physical exam, his weight is greater than the 95th percentile, and the right hip appears externally rotated with restriction of internal rotation, abduction, and flexion. Antero-posterior and frog-leg radiographs reveal widening of the right physis and a crescent-shaped area of increased density of the femoral neck. Which of the following is the most likely diagnosis? Developmental dysplasia of the hip Hip fracture Legg-Calve-Perthes disease Slipped capital femoral epiphysis

Correct Answer ( D ) Explanation: The boy has clinical and radiographic findings that are consistent with slipped capital femoral epiphysis (SCFE). SCFE involves failure of the physis and displacement of the femoral head relative to the neck that commonly affects adolescents aged 11 to 16 years. Acute SCFE presents with prodromal symptoms, occurring for less than three weeks. The patients complain of groin, thigh, or knee pain and may report a relatively minor injury, which is not sufficiently violent to produce an acute fracture of this severity. Chronic SCFE presents with a few-month history of vague pain and limp. Patients can have a limp with a lower extremity that is externally rotated. Acute-on-chronic SCFE presents with features of both conditions. Physical examination findings in SCFE show a restriction of internal rotation, abduction, and flexion. Diagnosis is made using the radiographic antero-posterior and frog-leg views. Findings include irregularity and widening of the physis, decrease in epiphyseal height, a crescent-shaped area of increased density in the proximal portion of the femoral neck, and a double density created from the anteriorly displaced femoral neck overlying the femoral head. Developmental dysplasia of the hip (A) is the abnormal position of the femoral head relative to the acetabulum commonly diagnosed in infants, and radiographic findings would show asymmetry of the femoral head ossification center and delayed ossification of the femoral head. Hip fracture (B) is a marker of high-energy trauma and is often associated with other musculoskeletal injuries. These were not evident of the vignette. Legg-Calve-Perthes disease (C) is the syndrome of idiopathic avascular necrosis of the femoral epiphysis showing fragmentation, sclerosis, and subchondral collapse of the ossification center of the femoral head on radiographs.

A 55-year-old man presents to the emergency department complaining of palpitations, nausea, and dizziness for 30 minutes. He is afebrile, his pulse is 140 beats per minute, and his blood pressure is 78/50 mm Hg. As the nurse attaches him to the cardiac monitor, you see the rhythm strip seen above. What is the most appropriate treatment? Adenosine 6 mg IV push Defibrillation at 360 joules Procainamide 18 mg/kg infused over 30 minutes Synchronized cardioversion at 200 joules

Correct Answer ( D ) Explanation: The monitor shows a wide complex tachycardia. Clinically, he is hypotensive and unstable, which necessitates the use of electrical rather than pharmacologic treatment. Synchronized cardioversion is indicated for the treatment of unstable tachydysrhythmias, including certain supraventricular dysrhythmias as well as monomorphic ventricular tachycardia. For treatment of ventricular fibrillation or polymorphic ventricular tachycardia, defibrillation (not synchronized cardioversion) is indicated. Adenosine (A) is a short-acting AV-nodal blocker used frequently in supraventricular dysrhythmias. Its use is not recommended in hypotensive patients with a wide-complex tachycardia. Defibrillating a patient who has a pulse (B) is dangerous and can result in the R-on-T phenomenon. This occurs when a depolarizing impulse (endogenous or exogenous) is delivered during ventricular repolarization (T wave). This can result in polymorphic ventricular tachycardia or ventricular fibrillation. To avoid this complication, synchronized cardioversion coordinates delivery of an electrical impulse so that it occurs with initiation of ventricular contraction (i.e., at the beginning of the QRS complex), thereby avoiding a shock during the relative refractory period of the cardiac cycle. Procainamide (C) is an option for patients with stable ventricular tachycardia, but it is not appropriate treatment of a hypotensive patient with an undifferentiated wide complex tachycardia. Procainamide may also worsen hypotension.

A 12-year-old boy presents with facial weakness for 2 hours. He states he woke up this morning and noticed that the right side of his face "doesn't move the right way." Examination reveals a right facial droop of the upper and lower face. He is unable to fully close his right eye. Examination is otherwise unremarkable. What management should be pursued? CT scan of the head Intravenous thrombolytics Lumbar puncture Prednisone

Correct Answer ( D ) Explanation: This patient presents with a peripheral facial nerve (CN VII) palsy or Bell's palsy, which should be managed with oral steroids (prednisone). Bell's palsy is an idiopathic, peripheral facial nerve palsy. The facial nerve innervates the muscles of the scalp, external ear and the muscles of facial expression. Additionally, the sensory portion supplies the anterior two thirds of the tongue. Patients will typically present with complaints of weakness of the facial muscles including the mouth, eyelid and forehead. One of the critical issues is to ensure that the paralysis is, in fact, peripheral and not central. In central facial nerve palsy, the muscles of the upper part of the face (forehead in particular) are spared. This is due to the fact that the upper third of the face has dual innervation; it receives fibers from the contralateral facial nerve. Thus, in a patient who has sparing of the upper part of the face, further diagnostic workup should be performed to determine the cause of central facial nerve paralysis. Peripheral facial nerve palsy is treated with prednisone 1 mg/kg/day for 7-10 days. The role of antivirals (i.e. acyclovir) is controversial as it is unclear if herpes simplex virus 1 is a causative organism. Additionally, artificial tears and taping the affected eye closed at night are recommended to prevent drying out of the cornea and subsequent abrasions or ulcerations. CT scan of the head (A) does not play a role as the lesion in Bell's palsy is outside of the central nervous system. Bell's palsy is not part of a stroke syndrome and thus should not be treated with thrombolytics (B). Peripheral facial nerve palsy does not result from central nervous system infection obviating the need for a lumbar puncture (C).

A 47-year-old man with a history of alcohol abuse presents with severe abdominal pain, nausea and vomiting for 1 day. Examination reveals marked epigastric tenderness to palpation. Labs show: Lipase: 4300; AST:451; ALT:532; Alk Phos:313; Total Bilirubin: 5.3. Which of the following is the most likely management indicated? CT scan of the abdomen and pelvis Discharge home if patient tolerates oral fluids Intravenous antibiotics and admission Right upper quadrant ultrasound

Correct Answer ( D ) Explanation: This patient presents with pancreatitis caused by gallstones. Pancreatitis is a complicated disease with a relatively high associated mortality rate of 4-7%. The most common cause of pancreatitis is gallstones with alcohol abuse as the number two cause in most populations. It is believed that biliary pancreatitis results from either a stone in the bile duct causing increased transmural pressure on the pancreatic duct or a stone in the common pancreatic and bile duct causes direct obstruction. Both causes result in activation of pancreatic enzymes within the pancreatic duct and pancreas leading to inflammation. The mechanism of alcoholic pancreatitis is not well understood. Laboratory studies can aid both in the diagnosis of pancreatitis as the cause of the patient's symptoms and in determining the cause of pancreatitis. Serum lipase evaluation represents the most commonly used test in diagnosis. Lipase levels increase within 4-8 hours of symptom onset and peak at 24 hours. The levels stay high for 8-14 days giving the test a good sensitivity. Additionally, the specificity ranges from 80-99%. In general, a cutoff of twice the upper limit of normal is used to reduce the number of patients missed. An elevation in the patient's liver enzymes (AST and ALT), alkaline phosphatase and bilirubin indicate a biliary etiology of pancreatitis. Confirmation should be obtained with an ultrasound of the right upper quadrant focused on the liver, gall bladder and biliary tree. Determination of a biliary cause of pancreatitis is important because endoscopic retrograde cholangiopancreatography (ERCP) removal of gallstones has been shown to decrease mortality and morbidity especially in patients with severe pancreatitis. CT scan of the abdomen and pelvis (A) is the optimal modality in the Emergency Department for imaging the pancreas. However, the accuracy of ultrasound for biliary disease is superior to that of CT scan. If gallstones are suspected to be responsible for disease, ultrasound is preferred. When gallstones are not suspected, CT scan is recommended in severe pancreatitis, acute deterioration, lack of improvement within 72 hours or the diagnosis is uncertain. Some patients with acute pancreatitis can be treated as outpatients (B) if they have mild disease, are tolerating oral fluids and have rapid outpatient follow up. In patients with moderate to severe disease, inpatient treatment is recommended. Antibiotics (C) have not been shown to improve outcomes in patients with pancreatitis regardless of disease severity.

A 23-year-old man presents with abdominal pain, vomiting and two loose, nonbloody stools. Physical examination reveals right lower quadrant tenderness to palpation. A CT is performed showing a normal appendix and some inflammation at the ileocecal junction. What pathogen is commonly implicated in this disorder? Aeromonas species Salmonella enterica Vibrio parahaemolyticus Yersinia enterocolitica

Correct Answer ( D ) Explanation: Yersinia enterocolitica infection can cause an ileocecitis, which may mimic the signs and symptoms of acute appendicitis. Y. enterocolitica infection results from oral ingestion of the bacteria with subsequent invasion of intestinal mucosa. About 2/3 of patients present with features of gastroenteritis - diarrhea, nausea, vomiting, colicky abdominal pain and fever. About 1/3 of patients will present with minimal or no diarrhea and a clinical presentation similar to appendicitis. Y. enterocolitica infection is typically self limited and requires supportive care only. Aeromonas species (A), Salmonella enterica (B), Vibrio parahaemolyticus (C) all cause a typical gastroenteritis picture but do not mimic appendicitis.


Conjuntos de estudio relacionados

Public Relations Exam 1 - Rainwater

View Set

Alcohol, Tobacco, and Other Drugs

View Set

Econ Midterm 1 (Ch 3), Money, Banking, and Econ, Econ Midterm 1 (Chapter 4), Econ Midterm 1 (Chapter 2), Econ Midterm 1 (Chapter 1), Econ Midterm 1 (Chapter 5)

View Set

Developmental and Genetic Influences on Child Health Promotion

View Set

Emotion, Stress, and Health - Midterm 2 (PSYC*3410)

View Set

Critical Care- Wk 6 Quiz/Wk 7 Exam

View Set